20.08.2015 Views

Massachusetts Tests for Educator Licensure (MTEL ) www.mtel.nesinc.com

Massachusetts Tests for Educator Licensure (MTEL ) www.mtel ...

Massachusetts Tests for Educator Licensure (MTEL ) www.mtel ...

SHOW MORE
SHOW LESS
  • No tags were found...

Create successful ePaper yourself

Turn your PDF publications into a flip-book with our unique Google optimized e-Paper software.

<strong>Massachusetts</strong><strong>Tests</strong> <strong>for</strong> <strong>Educator</strong> <strong>Licensure</strong> ® (<strong>MTEL</strong> ® )<strong>www</strong>.<strong>mtel</strong>.<strong>nesinc</strong>.<strong>com</strong>Copyright © 2009 Pearson Education, Inc. or its affiliate(s). All rights reserved.Evaluation Systems, Pearson, P.O. Box 226, Amherst, MA 01004<strong>Massachusetts</strong> <strong>Tests</strong> <strong>for</strong> <strong>Educator</strong> <strong>Licensure</strong> and <strong>MTEL</strong> are trademarks, in the U.S. and/or other countries, of the<strong>Massachusetts</strong> Department of Elementary and Secondary Education and Pearson Education, Inc. or its affiliate(s).Pearson and its logo are trademarks in the U.S. and/or other countries of Pearson Education, Inc. or its affiliate(s).01/10


Spanish (28) Practice TestINTRODUCTIONThis document is a printable version of the <strong>Massachusetts</strong> <strong>Tests</strong> <strong>for</strong> <strong>Educator</strong> <strong>Licensure</strong>® (<strong>MTEL</strong>®) Spanish (28)Online Practice Test. This practice test is a sample test consisting of 100 multiple-choice questions and 2 openresponseitem assignments.To assist you in recording and evaluating your responses on the practice test, a Multiple-Choice Answer Sheet, anAnswer Key Worksheet, and an Evaluation Chart by test objective are included <strong>for</strong> the multiple-choice questions.A blank Response Sheet, Evaluation In<strong>for</strong>mation, and Sample Responses and Analyses, as well as a ScoringRubric, are included <strong>for</strong> the open-response items. Lastly, there is a Practice Test Score Calculation worksheet.PURPOSE OF THE PRACTICE TESTThe practice test is designed to provide an additional resource to help you effectively prepare <strong>for</strong> the <strong>MTEL</strong>Spanish (28) test. The primary purpose of the practice test is to help you be<strong>com</strong>e familiar with the structure andcontent of the test. It is also intended to help you identify areas in which to focus your studies. Education facultyand administrators of teacher preparation programs may also find this practice test useful as they help studentsprepare <strong>for</strong> the official test.TAKING THE PRACTICE TESTIn order to maximize the benefits of the practice test, it is re<strong>com</strong>mended that you take this test under conditionssimilar to the conditions under which the official <strong>MTEL</strong> tests are administered. Try to take the practice test in aquiet atmosphere with few interruptions and limit yourself to the four-hour time period allotted <strong>for</strong> the official testadministration. You will find your results to be more useful if you refer to the answer key only after you have<strong>com</strong>pleted the practice test.INCORPORATING THE PRACTICE TEST IN YOUR STUDY PLANAlthough the primary means of preparing <strong>for</strong> the <strong>MTEL</strong> is your college education, adequate preparation prior totaking or retaking the <strong>MTEL</strong> test is strongly re<strong>com</strong>mended. How much preparation and study you need dependson how <strong>com</strong><strong>for</strong>table and knowledgeable you are with the content of the test.The first step in preparing to take the <strong>MTEL</strong> is to identify what in<strong>for</strong>mation the test will address by reviewing theobjectives <strong>for</strong> your field. A <strong>com</strong>plete, up-to-date list of the Test Objectives is included in the Test In<strong>for</strong>mationBooklet <strong>for</strong> each test field. The test objectives are the core of the testing program and a helpful study tool.Be<strong>for</strong>e taking or retaking the official test, focus your study time on those objectives <strong>for</strong> which you wish tostrengthen your knowledge.This practice test may be used as one indicator of potential strengths and weaknesses in your knowledge of thecontent on the official test. However, because of potential differences in <strong>for</strong>mat and difficulty between thepractice test and an official <strong>MTEL</strong> Spanish (28) test, it is not possible to predict precisely how you might score onan official <strong>MTEL</strong> Spanish (28) test. Keep in mind that the subareas <strong>for</strong> which the test weighting is greatest willreceive emphasis on this test. Refer to the Test In<strong>for</strong>mation Booklet <strong>for</strong> additional in<strong>for</strong>mation about how toprepare <strong>for</strong> the test.1


Spanish (28) Practice TestSPANISHPRACTICE TEST2


Spanish (28) Practice TestGENERAL TEST DIRECTIONSThis practice test consists of four sections: (1) a listening section with multiple-choice questions, (2) a readingsection with multiple-choice questions, (3) a writing assignment, and (4) a speaking assignment. Each multiplechoicequestion on the practice test has four answer choices. Read each question carefully and choose the ONEbest answer. Record each answer on the answer sheet provided.Sample Question: 1. What is the capital of <strong>Massachusetts</strong>?A. WorcesterB. New Bed<strong>for</strong>dC. BostonD. SpringfieldThe correct answer to this question is C. You would indicate that on the answer sheet.Directions <strong>for</strong> the writing and speaking item assignments appear immediately be<strong>for</strong>e those assignments.You may wish to monitor how long it takes you to <strong>com</strong>plete the practice test. When taking the actual <strong>MTEL</strong>Spanish (28) test, you will have one four-hour test session in which to <strong>com</strong>plete the test. The section of listeningmultiple-choice questions will be administered first, and the speaking assignment will generally be administeredlast. However, you will be allowed to work on the multiple-choice questions in the reading section and thewriting assignment in any order that you choose.3


Spanish (28) Practice Test4MULTIPLE-CHOICE ANSWER SHEETQuestionNumberYourResponse12345678910111213141516171819202122232425262728293031323334QuestionNumberYourResponse35363738394041424344454647484950515253545556575859606162636465666768QuestionNumberYourResponse69707172737475767778798081828384858687888990919293949596979899100


Spanish (28) Practice TestDIRECTIONS FOR SECTION ONE: LISTENING COMPREHENSION(You will see in the test booklet and hear on the recording:)The first section of this test contains questions that involve listening to a recording. Each listening passage willbegin with directions that will tell you what to listen <strong>for</strong>. The directions will be read aloud on the recording.They are also printed in your test booklet. Listen carefully to the recording and follow along in your test booklet.Ample time is provided between questions <strong>for</strong> your responses. Record your answer on the answer sheet in thespace that corresponds to the question number. Unless otherwise indicated, each listening passage will be readtwice.Once the recording has begun, it cannot be stopped, nor can any part of it be replayed. There<strong>for</strong>e, listen carefully.If you have any questions or if you cannot hear this recording clearly, please raise your hand now.5


Spanish (28) Practice TestMULTIPLE-CHOICE QUESTIONS: LISTENING COMPREHENSION(You will see in the test booklet and hearon the recording:)1. Escuche atentamente el diálogo quesigue. Escuchará el diálogo dos veces.(You will hear on the recording:)male: Me siento muy orgulloso de ti,Carlita. Tus noticias me dan muchaalegría, te quiero felicitar de todo corazón.female: Gracias, papi. Nunca me habíaimaginado que este día llegaría tan pronto.Pensaba pasar más años a tu lado oyendotus buenos consejos. Te voy a extrañarmuchísimo.male: Bueno, pero lo mejor es que no vana estar lejos, y cuando regresen de la lunade miel, aquí les vamos a esperar con lacena de bienvenida.female: Te agradezco que hayas aceptadoa mi <strong>com</strong>pañero de vida con los brazosabiertos. No es siempre así.Ahora conteste la pregunta.(You will see in the test booklet:)Según el diálogo, ¿sobre qué evento estáhablando Carla con su padre?A. el casamiento con su novioB. su graduación de la universidadC. el nacimiento de su hijoD. su mudanza a otra ciudad6


Spanish (28) Practice Test(You will see in the test booklet and hearon the recording:)Escuche atentamente el pasaje quesigue; luego conteste las dos preguntas acontinuación. Escuchará el pasaje dosveces.(You will hear on the recording:)¡Buenos días! La "operación salida" delverano <strong>com</strong>enzó esta mañana y un añomás el in<strong>for</strong>me diario de la DirecciónGeneral de Tráfico nos advierte de fuertescongestiones en las principales vías desalida de Madrid que se extienden a lasplayas mediterráneas. Si bien todas lasautopistas radiales sufren atascos, es en lacarretera de la Coruña y en la carretera deValencia, donde se localizan los mayoresparones. Por eso, la carretera de Castillaes una opción para evitar la congestión enese tramo. Es posible que se demore eltráfico unos 10 kilómetros fuera de lasplayas donde se construye otra salida parauna nueva autopista de peaje.Ahora conteste las dos preguntas.(You will see in the test booklet:)2. En este pasaje, se puede deducir que:A. estos atascos se producendiariamente.B. es la primera vez que se producenestos atascos.C. estos atascos se producenanualmente, en época estival.D. los atascos se producen solo cuandohay un in<strong>for</strong>me.3. Según el pasaje, para eludir el atasco en lacarretera de la Coruña, el locutor proponeque se tome:A. las calles principales a la playa.B. la carretera de Valencia.C. la carretera de Castilla.D. la nueva autopista de peaje.7


Spanish (28) Practice Test(You will see in the test booklet and hearon the recording:)Escuche atentamente el diálogo quesigue; luego conteste las dos preguntas acontinuación. Escuchará el diálogo dosveces.(You will hear on the recording:)male: Paulina, ¿vas a asistir al seminarioen octubre? Como somos miembros deesta organización de educadores, tenemosderecho a un descuento por inscribirnostemprano. A lo mejor, terminamos siendocolegas en esta escuela, sería una granoportunidad para estar con otra personaconocida.female: Pepe, no estoy segura de siquisiera asistir este año – sería la cuartavez que he pasado el otoño en unseminario. Me gustaría explorar otrasopciones profesionales para el próximoverano pero esta ganga puedeconvencerme.male: Tengo el anuncio en mi oficinay podemos confirmar la fecha paraconseguir esta rebaja.Ahora conteste las dos preguntas.(You will see in the test booklet:)4. ¿Cuál de las siguientes opciones es la ideaprincipal de este diálogo?A. conversar sobre planes profesionalespara el próximo veranoB. explorar un descuento máximo porestar afiliados de esta organizaciónC. beneficiarse por inscribirsetemprano en el seminarioD. decidir qué seminarios ofrece estaorganización5. En este diálogo se puede asumir que:A. Paulina no va a este seminario portener otras prioridades inmediatas.B. Pepe tiene que convencer a Paulinapara que asista.C. Paulina prefiere viajar en el otoño.D. Pepe nunca ha asistido solo a otrasconferencias en el pasado.8


Spanish (28) Practice Test(You will see in the test booklet and hearon the recording:)Escuche atentamente el diálogo quesigue; luego conteste las dos preguntas acontinuación. Escuchará el diálogo dosveces.(You will hear on the recording:)female: Hola Ricardo. ¿Qué haces aquíen la sección de libros de cocina?male: Ay, Tamia, tratando de bregar conun problema alimenticio y culinario. Túsabes cómo me gustan los granos, pero seme ha salido una alergia al trigo, y lacebada me está cayendo pesadísima.female: ¡Qué suerte tienes que yo seaboliviana! . . . porque la gente de mi tierratiene una solución nutritiva y económica atu dilema.male: Bueno, dime, porque ya me estoycansando de arroz y más arroz.female: Es un grano, cuyos orígenes sondel altiplano, la quínua. Aunque tiene eltamaño de una semilla de sésamo, tiene unvalor nutritivo más alto que cualquier otrograno.male: Te creo todo eso, . . . pero lo másseguro es que sepa a paja o a aserrín.(You will see in the test booklet:)6. ¿Cuál de los siguientes granos lere<strong>com</strong>ienda la muchacha al amigo?A. el trigoB. la cebadaC. el arrozD. la quínua7. Según el diálogo, este grano puede ser útilporque ahora:A. se ofrece un alimento alternativo alos que son alérgicos a <strong>com</strong>idastradicionales.B. los bolivianos pueden preparar losmismos platos que se preparanusando granos tradicionales.C. se reconoce <strong>com</strong>o una buena fuentede proteína <strong>com</strong>pleta para losvegetarianos.D. los bolivianos pueden cultivar estegrano en el altiplano en mayorcantidad.female: No, no; es deliciosa, y puedescocinar con ella cualquier plato que hagascon cualquier otro grano. Puedes hacerpilau, ensaladas, sopas y hasta postres.Ahora conteste las dos preguntas.9


Spanish (28) Practice Test(You will see in the test booklet and hearon the recording:)Escuche atentamente el diálogo quesigue; luego conteste las dos preguntas acontinuación. Escuchará el diálogo dosveces.(You will hear on the recording:)female: Buenas tardes, señor. Acabo deescuchar el anuncio que encontraron unacartera en el almacén. Es probable quesea la mía. La mía tiene mis iniciales,JMG.male: Sí, claro, señora. ¿Puede decirmepor qué piensa que la cartera es la suya?female: Estaba en el tercer piso delalmacén y quería devolver una bufandaque no me gustó y cambiarla por otra.Por eso necesitaba mostrar una copiadel recibo que estaba en mi cartera.Al cambiarla, firmé otro recibo aldependiente. Me apuré y decidí ponertodas mis <strong>com</strong>pras en una bolsa. Estoysegura que dejé mi cartera en elmostrador.male: ¿Puede identificar algo más de sucartera?(You will see in the test booklet:)8. Según el diálogo, ¿cuál de las siguientesaseveraciones es cierta sobre la cartera dela mujer?A. Contiene dos tarjetas de crédito.B. Se la dejó en la oficina.C. Se la dejó en una bolsa.D. Contiene el recibo de la receta.9. Según el diálogo, ¿cuál de las siguientesopciones es más apropiada para <strong>com</strong>enzaresta conversación?A. "Señora, ya hemos cerrado laoficina."B. "Señora, ¡Qué suerte! Todo está enoferta."C. "Señora, tenemos carteras de todoslos colores."D. "Señora, ¿en qué puedo servirle?"female: Déjeme ver . . . había 50 eurosen efectivo, dos tarjetas de crédito, milicencia de conducir y una receta nuevapara mi alergia.male: Bueno, esta cartera esdefinitivamente la suya. Me alegrode que se la hayamos podido devolver.female: Gracias.Ahora conteste las dos preguntas.10


Spanish (28) Practice Test(You will see in the test booklet and hearon the recording:)Escuche atentamente el diálogo quesigue; luego conteste las dos preguntas acontinuación. Escuchará el diálogo dosveces.(You will hear on the recording:)male: Isabel, ya nos estamos acercandoal área donde nos dijeron que estaba laestación.female: Paco, mira el mapa y dime sipuedas encontrar la calle San Antonio.Ahora estamos en la Avenida Robles y laentrada de la estación de trenes deberíaestar en la mano izquierda en esa calle.male: Isabel, no la veo y tampoco lacalle. ¿Estás segura de que la estaciónqueda en la calle San Antonio?Ahora conteste las dos preguntas.(You will see in the test booklet:)10. ¿Cuál de las siguientes aseveracionesdescribe mejor la idea principal de estediálogo?A. el leer un horario de trenesB. el conducir en la Avenida RoblesC. el procurar un mapaD. el encontrar dónde está la calle SanAntonio11. Se puede inferir que la relación entre loshablantes es:A. una conductora de autobuses y unpasajero.B. una turista y un taxista.C. dos amigos.D. una mujer policía y un peatónperdido.11


Spanish (28) Practice Test(You will see in the test booklet and hearon the recording:)Escuche atentamente el diálogo quesigue; luego conteste las dos preguntas acontinuación. Escuchará el diálogo dosveces.(You will hear on the recording:)male: ¿Y adónde vas a ir de vacacioneseste verano?female: Mmmmm . . . no sé. La verdades que quiero unas vacaciones diferentes.male: ¿Cómo vas a saber? Has viajadotanto que ya no tienes sueños acerca delugares que no hayas visitado.female: ¡Mentira! Mira: me gustaríavisitar un lugar que no sea muy turístico,tranquilo y que no tenga que visitar nimuseos ni monumentos históricos.male: Eso es fácil, vete a un lugar quetenga playa.female: No sé, quiero ver algo más quemar y arena. ¿Podrías ayudarme aencontrar algo que responda a mis deseos?Ahora conteste las dos preguntas.(You will see in the test booklet:)12. Según el diálogo, ¿cuál de las siguientesopciones es la respuesta más apropiada alo que dice la mujer?A. Desde luego, la ciudad ofrecemuchas cosas y te va a gustar.B. Por supuesto, conozco un lugartranquilo y con actividades deecoturismo.C. Ay, claro, yo viajaré contigo porqueme encantan los museos.D. Bueno, gracias por la invitación.Nos vamos a divertir mucho en lasvacaciones.13. ¿Cuál de las siguientes opciones mejordescribe el tono de la mujer en estediálogo?A. deprimidoB. nerviosoC. dudosoD. preocupado12


Spanish (28) Practice Test(You will see in the test booklet and hearon the recording:)Escuche atentamente el pasaje quesigue; luego conteste las dos preguntas acontinuación. Escuchará el pasaje dosveces.(You will hear on the recording:)No es fácil estar al día con las demandaslaborales y familiares y, además, encontrartiempo para uno mismo. La vida parecefuera de control y la tranquilidad mentales un concepto <strong>for</strong>áneo. Sin embargo, esposible lograr un equilibrio razonableentre el trabajo, la familia y uno mismo.Por supuesto, habrán días en que elequilibrio se altere, eso es parte normalde la vida. Lo importante, entonces,es detenerse a pensar qué cosas sonimportantes y cómo darle a esas cosas eltiempo que merecen. De esta manera,usted podrá tener más tiempo para susseres queridos y para actividades que seansatisfactorias y divertidas.Ahora conteste las dos preguntas.(You will see in the test booklet:)14. Según el pasaje, ¿cuál de las siguientesaseveraciones es el primer paso paraencontrar un equilibrio entre las demandaslaborales y familiares?A. poner control a la vida personalB. buscar la paz mental, tanto personal<strong>com</strong>o la de toda la familiaC. dar prioridad a las cosas que sonmás importantesD. sacar más tiempo para estar con losseres queridos15. Es más probable que se dirija estepasaje a:A. estudiantes de secundaria.B. personas jubiladas.C. personas que trabajan.D. estudiantes universitarios.13


Spanish (28) Practice Test(You will see in the test booklet and hearon the recording:)16. Escuche atentamente el pasaje quesigue. Escuchará el pasaje dos veces.(You will hear on the recording:)Cuando era niña, siempre íbamos devacaciones a una isla con toda la familia ycon mis abuelos también. Esos recuerdosme traen a la memoria los momentos másfelices que pasé con mi hermano y mihermana, con quienes me divertía muchoen la playa o en casa inventando juegosy actividades para pasar el tiempo. Erala única oportunidad de conocer a misprimos, que vivían lejos de nosotros yvenían cada dos años para <strong>com</strong>partirun tiempito con nuestros abuelos y connosotros. Durante treinta años, he visitadola casa que construyeron mis abuelos devez en cuando, a veces muy a menudo ya veces con años entre visitas. Hace pocose murieron mis abuelos, y no podemosdecidir qué hacer con la casa. Ahoratodos mis hermanos y primos viven lejosy casi no la visitan. Pero todavía tiene unsignificado muy importante para todos.(You will see in the test booklet:)La hablante tiene tantos sentimientos porla casa de sus abuelos porque:A. tiene buenas memorias de su niñezallí.B. le gusta reunirse con sus primos ysus hermanos allí.C. le gusta visitar a sus abuelos en lacasa.D. visita la casa de sus abuelos conmucha frecuencia.Ahora conteste la pregunta.14


Spanish (28) Practice Test(You will see in the test booklet and hearon the recording:)17. Escuche atentamente el pasaje quesigue. Escuchará el pasaje dos veces.(You will hear on the recording:)Gracias por llamar a la oficina deadmisiones. Este mensaje incluyein<strong>for</strong>mación que le ayudará en el procesode solicitud de ingreso a la universidad.Primero, es importante que relleneel <strong>for</strong>mulario por <strong>com</strong>pleto. No seprocesarán solicitudes que no incluyantoda la in<strong>for</strong>mación requerida. Segundo,la clínica de la universidad deberá recibircopias de todos sus datos personalesy el nombre de su médico personal.Finalmente, cuando hayamos recibidotodo, nuestra oficina le llamará paracoordinar su visita al campus, y unaentrevista con un representante de laoficina de admisiones. Después de laentrevista, nuestra oficina se <strong>com</strong>unicarácon usted en tres semanas. Si loaceptamos <strong>com</strong>o estudiante, es importanteque envíe el depósito de su matrícula anuestra oficina lo más pronto posible paraasegurar su ingreso. Si tiene preguntasadicionales, por favor permanezca en lalínea hasta que uno de nuestros asistenteslo atienda.(You will see in the test booklet:)Según el pasaje, le va a llamar la oficinade admisiones para concertar una visitacuando:A. hayan recibido el <strong>for</strong>mulario.B. se hayan satisfecho todos losrequisitos.C. haya sido recibido el depósito.D. la oficina se haya <strong>com</strong>unicado conel médico.Ahora conteste la pregunta.15


Spanish (28) Practice Test(You will see in the test booklet and hearon the recording:)Escuche atentamente el diálogo quesigue; luego conteste las dos preguntas acontinuación. Escuchará el diálogo dosveces.(You will hear on the recording:)female: ¡Apúrate, Adolfo! Nuestrareservación es a las siete y media, y yason las siete.male: Ya lo sé, Marcela, y te pidodisculpas.female: ¿En cuánto tiempo estarás listo?Nuestros hijos ya están esperándonos.male: En unos quince minutos. ¿Por quéno llamas a ver si pueden mantener lareservación hasta las ocho?Ahora conteste las dos preguntas.(You will see in the test booklet:)18. ¿Cuál de las siguientes palabras mejordescribe la relación entre los hablantes?A. amigosB. hermanosC. espososD. colegas19. Según el diálogo, se infiere que Adolfo yMarcela podrían:A. pedir una mesa más grande en esterestaurante.B. conseguir su reservación esta noche.C. llamar al otro restaurante para pediruna reservación esta noche.D. perder una mesa en este restaurante.16


Spanish (28) Practice Test(You will see in the test booklet and hearon the recording:)Escuche atentamente el pasaje quesigue; luego conteste las dos preguntas acontinuación. Escuchará el pasaje dosveces.(You will hear on the recording:)¡Atención! Por favor, no se estacioneni deje su vehículo solo delante de laterminal. Esta es zona limitada, sólopara dejar pasajeros que necesitan ir almostrador, o a aquéllos que tienen sustarjetas de embarque y desean despacharsu equipaje. Si desea pasar al área deespera, proceda al primer nivel, junto alárea de entrega de equipajes.Ahora conteste las dos preguntas.(You will see in the test booklet:)20. Según el pasaje, la razón más probable porla cual los pasajeros pueden despachar suequipaje al borde de la acera es que:A. tienen cómodo acceso a la terminal.B. deben despachar su equipaje antesde acercarse al mostrador.C. consiguen sus tarjetas de embarqueantes de entrar a la terminal.D. ya tienen sus tarjetas de embarque.21. ¿Cuál de las siguientes oraciones es másprobable que sea una de las intencionesdel locutor en este anuncio?A. dar instrucciones de cómo llegar a laterminalB. aconsejar sobre las zonas deestacionamientoC. hacer una advertencia sobre losvehículos desatendidosD. in<strong>for</strong>mar cómo recuperar el equipajeen el área de entrega17


Spanish (28) Practice Test(You will see in the test booklet and hearon the recording:)22. Escuche atentamente el pasaje quesigue. Escuchará el pasaje dos veces.(You will hear on the recording:)¡Hola! En este momento, no nosencontramos en la oficina, pero nosimporta mucho responder a sus preguntas.Por favor, escuche las siguientes opcionespara dejar su mensaje en el buzónapropiado, y nos <strong>com</strong>unicaremos conusted lo más pronto posible. Para haceruna cita con un médico, pulse el número1; para solicitar una receta en la farmacia,pulse el número 2; para una referencia aun especialista, pulse el número 3; parapreguntas sobre su seguro o relacionadascon su cuenta, pulse el número 4; paraotros asuntos, pulse el número 5. Graciaspor su paciencia y que pase un buen día.Ahora conteste la pregunta.(You will see in the test booklet:)Probablemente se escucha estemensaje en:A. una farmacia.B. un dentista.C. un veterinario.D. una clínica.(You will see in the test booklet and hearon the recording:)23. Escuche atentamente el diálogo quesigue. Escuchará el diálogo dos veces.(You will hear on the recording:)male: Paco no se ha levantado estamañana, Adriana. Sabes que él tiene quetomar un examen en su clase de historia enuna hora y no puede perderlo. Ahora, noserá posible que él tome el autobús porqueel último salió hace 20 minutos.female: Manolo, él me ignoró cuandotraté de despertarlo. Se acostó muy tardeanoche, dando prioridad a sus videojuegosen vez de estudiar. Además, lo conocesbien, siempre aprovecha dormir lo mástarde posible.male: ¡Claro que sí lo conozco bien!¡Más que nada me parece que Pacoprefiere hacerse el dormido en vez dellegar a tiempo para su examen dehistoria!Ahora conteste la pregunta.(You will see in the test booklet:)Según el diálogo, ¿qué significa la frasehacerse el dormido?A. dormir bienB. pretender estar dormidoC. tener insomnioD. caminar sonámbulo18


Spanish (28) Practice Test(You will see in the test booklet and hearon the recording:)Escuche atentamente el diálogo quesigue; luego conteste las tres preguntasa continuación. Escuchará el diálogodos veces.(You will hear on the recording:)female: Ay, Eduardo. ¡Qué suerte quenos encontremos esta tarde! Estaba portelefonearte para . . .male: Bueno, Rosalía. Estoy contentode verte también. Sabes, tengo buenasnoticias de la solicitud que entregué.Me ofrecieron una entrevista.female: Sí, claro, son buenas noticias.¡Felicitaciones! Pero, ¿recuerdas losfolletos de la <strong>com</strong>pañía que te presté antesde que entregaras la solicitud?male: Por supuesto, pero no puedodiscutir esos detalles mientras esté tanalegre por mis buenas noticias.female: Eduardo, por favor, los necesitoahorita porque esta semana yo voy asolicitar un puesto administrativo allátambién. ¿Cuándo puedo contar conellos? Me parece que estoy esperando aque las ranas críen pelo.male: Está bien, está bien. Te losdevuelvo en cuanto pueda.Ahora conteste las tres preguntas.(You will see in the test booklet:)24. ¿Cuál de las siguientes opciones explicamejor la frase esperar a que las ranascríen pelo?A. tener éxitoB. llegar tardeC. esperar muchoD. estar preocupado25. ¿Cuál de las siguientes oraciones describemejor la actitud de Eduardo a Rosalía enla conversación?A. Se interesa por los pensamientosde ella.B. No presta atención a lo que diceella.C. Expresa indiferencia hacia laspreguntas de ella.D. Se siente molesto porque ella no lollamó.26. Cuando Eduardo le devuelva los folletos aRosalía, ella podrá:A. obtener un puesto en esta <strong>com</strong>pañía.B. conseguir una entrevista en unasemana.C. saber más detalles de la <strong>com</strong>pañía.D. aceptar su nuevo puestoadministrativo.END OF LISTENING SECTION19Please go on.


Spanish (28) Practice TestDIRECTIONS FOR SECTION TWO: READING COMPREHENSIONSection Two is a reading section with multiple-choice questions, each with four answer choices. Read eachquestion carefully and choose the ONE best answer. Record your answer on the answer sheet in the space thatcorresponds to the question number.20


Spanish (28) Practice TestMULTIPLE-CHOICE QUESTIONS: READING COMPREHENSIONLea atentamente el pasaje que sigue; luego conteste las dos preguntas a continuación.Buñol, España, es un pueblo con más de 50 mil años de historia de ocupaciones romanas, árabes yfrancesas. Sin embargo, hoy en día se conoce por La Tomatina, celebración de poco más de 60 añosde tradición. A fines de agosto <strong>com</strong>ienzan a llegar al pueblo de 9.000 habitantes una invasión de unos30 o 40 mil turistas. Las 200 toneladas de tomates muy maduros que se necesitan para esta batalla deverduras de una hora de duración las provee el ayuntamiento. En 1945, durante un desfile <strong>com</strong>unal sedesató una batalla campal cuyos proyectiles fueron las hortalizas de los puestos del mercado. El episodiofue tan memorable que desde entonces se celebra anualmente La Tomatina en la plaza del pueblo, y milesde personas luchan con proyectiles de tomates muy maduros y aplastados hasta que las autoridades dan laorden de cese.27. Según el pasaje, La Tomatina <strong>com</strong>enzóa celebrarse hace:A. más de 50 mil años.B. 200 años.C. poco más de 60 años.D. 9.000 años.28. La ironía de la situación en el pasajees que:A. un pueblo con tanta tradiciónhistórica se conozca por LaTomatina.B. vengan tantos turistas a un pueblotan pequeño.C. La Tomatina se celebre anualmenteen la plaza del pueblo.D. en un pueblo pacífico se festeje unalucha.21


Spanish (28) Practice TestLea atentamente el pasaje que sigue; luego conteste las tres preguntas a continuación.14 de marzo 2008Señora Laura MendozaPlaza Mayor #5, puerta 2ALima, PerúEstimada Sra. Mendoza:Hemos concluido con el proceso de revisión del Programa de Becas de la Universidad Central. Lamento<strong>com</strong>unicarle que no le hemos otorgado la beca a la que usted se postuló. Sin embargo, el Comité quiere<strong>com</strong>unicarle lo mucho que ha apreciado su solicitud.Tan pronto <strong>com</strong>o obtenga su título, esperamos que consulte las ofertas laborales de nuestras diferentessedes. Nos gustaría, además, <strong>com</strong>partir sus intereses en las áreas de enseñanza e investigación con<strong>com</strong>ités de búsqueda en áreas relacionadas a la suya.Todos los miembros del Comité queremos agradecerle el habernos dado la oportunidad de conocer suinvestigación. Le deseamos éxito en su trabajo académico.Atentamente,Luis MéndezDirector del Consejo de Investigaciones29. ¿Cuál es el propósito principal de estacarta?A. animar a la Sra. Mendoza a solicitaruna beca el próximo añoB. felicitar a la Sra. Mendoza por sucalidad académicaC. <strong>com</strong>unicar la negación de una becasolicitada por la Sra. MendozaD. in<strong>for</strong>mar a la Sra. Mendoza sobreposibles ofertas de trabajo22


Spanish (28) Practice Test30. Es más probable que una consecuencia deque la Sra. Mendoza termine su carrerauniversitaria sea que:A. la hagan miembro del <strong>com</strong>ité derevisión de programas de becas.B. tenga más oportunidades de empleoen esa agencia.C. tenga más posibilidades de que leden esa beca.31. ¿Cuál de los siguientes adjetivos mejordescribe el tono de esta carta?A. coloquialB. in<strong>for</strong>malC. <strong>for</strong>malD. jocosoD. le otorguen una beca en otra área deinvestigación.23


Spanish (28) Practice TestLea atentamente el pasaje que sigue; luego conteste las dos preguntas a continuación.A: Nuestros ClientesDe:Martín@verdesi.<strong>com</strong>Tema: Una Invitación PersonalLa Conferencia sobre Tecnologías Ecológicas está dirigida a las empresas y personas que esténinteresadas en aumentar la eficacia energética y reducir el consumo de energía. Si les interesa asistir, porfavor envíennos su nombre y dirección. Por primera vez, un aliciente: la entrada es gratuita si se inscribepor la red. Pueden hacerlo desde <strong>www</strong>.verdesi.<strong>com</strong>/conferencia/gratuita.html. ¡Pueden estar seguros deque no daremos su in<strong>for</strong>mación a otras empresas!La conferencia está planeada con fines educativos y en ella se exponen varios aspectos del tema de laecología. Este año, se centrará en la eficacia de las "energías verdes", la producción de la "energíaverde", las inversiones corporativas y las iniciativas gubernamentales. Otros seminarios que continúanlos temas del año pasado son el papel del viento en las tecnologías verdes, las energías renovables, lahuella de carbón, el reciclaje y los bio<strong>com</strong>bustibles. Además, hemos agregado exhibiciones detecnologías del año pasado para reducir al mínimo la producción de basura o agentes contaminantes.¡Esperamos verlos a todos ustedes!Nicolás MartínezMartín@verdesi.<strong>com</strong>32. ¿Cuál de las siguientes opcionesrepresenta uno de los seminarios nuevoseste año en la conferencia?A. la disminución de contaminantes delmedio ambienteB. el reciclaje corporativoC. el viento <strong>com</strong>o energía renovableD. las iniciativas ecológicasgubernamentales33. ¿Cuál de las siguientes opciones describemejor la intención del autor al ofrecer unincentivo?A. proporcionar la posibilidad dematricularse en personaB. asegurar que un público más amplioreciba el anuncioC. atraer un mayor número departicipantes este añoD. demostrar la disponibilidad depáginas Web para inscribirse24


Spanish (28) Practice TestLea atentamente el pasaje que sigue; luego conteste las tres preguntas a continuación.Los huevos son un alimento de alto contenido nutritivo, lo que significa que éstos proveen una altaproporción de las necesidades diarias de nutrientes a la vez que constituyen una baja proporción de lasnecesidades calóricas. Son una fuente excelente de proteína de alta calidad. De hecho, los huevos sonuno de los pocos alimentos que dan fuentes naturales de vitamina D. El color de la cáscara no tiene nadaque ver con el valor nutritivo, calidad o sabor del huevo. Las gallinas de diferentes razas ponen huevosde diferentes colores. Un huevo grande constituye el 10% del valor diario basado en una dieta de 2.000calorías. Debido a su alta cantidad de proteína, los huevos están clasificados en la Guía de la PirámideAlimenticia en la misma categoría que la carne roja, las aves, el pescado, los frijoles secos y las nueces.La yema y la clara tienen muchos minerales nutritivos que se necesitan consumir a diario e incluyen elfós<strong>for</strong>o, el hierro, el cobre, el magnesio, etc. La yema contiene toda la grasa, el colesterol y el 44% de laproteína del huevo.Además de ser una fuente de nutrición para todas las personas de todas las edades, los huevos no soncaros, son fáciles de adquirir y fáciles de preparar. Sin embargo, hay que conservar los huevos concuidado. Es necesario almacenar los huevos en el refrigerador hasta cocinarlos. Aunque se los cocinebien, no se debe arruinar su valor nutritivo cocinándolos con grasa adicional. Es preferible hervirlos envez de freírlos.34. Según este pasaje, se puede resumir quelos huevos tienen:A. todas las vitaminas naturales.B. proteína de alta calidad que leprovee toda la cantidad requerida adiario.C. muchas sustancias nutritivas.D. más proteína <strong>com</strong>pleja que la carneroja, el pescado, las aves y losfrijoles.36. ¿Cuál de las siguientes aseveraciones es elpunto de vista del autor en este pasaje?A. Un huevo contiene toda la grasanecesaria en una dieta diaria.B. El sabor de un huevo es el mismoindependientemente del color de lacáscara.C. El sabor y el color mejoran al hervirun huevo.D. Un huevo provee toda la cantidadrequerida de proteínas <strong>com</strong>plejas.35. Según este pasaje, ¿cuál de las siguientesacciones disminuye los beneficios para lasalud de los huevos?A. cocinarlos añadiendo grasaB. <strong>com</strong>erlos con otros tipos de proteínaC. conservarlos en el refrigeradorD. <strong>com</strong>er solamente la yema25


Spanish (28) Practice TestLea atentamente el pasaje que sigue; luego conteste las dos preguntas a continuación.Señor Jorge BustamanteCalle Sandoval, 28Valparaíso, ChileEstimado Señor Bustamante:Como Ud. ha demostrado ser muy buen cliente, quisiéramos hacerle una oferta especial. Le ofrecemosun teléfono gratis (tras el reembolso por correo) al activar una línea de servicio adicional en un plan dellamadas calificado. Esta es una excelente oferta ¡y sólo tiene que llamar al número indicado abajo!Este teléfono le encantará y viene con las últimas tecnologías:• Mensajes de texto• Cámara digital• Correo electrónico• Juegos y tonos agradables• Uso sin límite durante la noche y el fin de semanaAcepte su teléfono con cámara llamando al 1-800- 422-1272. No se demore, llame hoy mismo pararecibir todo.Atentamente,Mercedes JiménezDirectora Ejecutiva37. Según el pasaje, el señor Bustamenterecibió esta carta de Mercedes Jiménezporque él:A. solía enviar mensajes electrónicospor teléfono.B. ya tenía una cuenta telefónicacorriente con esta <strong>com</strong>pañía.C. había expresado interés por recibirofertas especiales de esta <strong>com</strong>pañía.D. quería conseguir otro teléfono conmuchas funciones.38. Según el pasaje, el señor Bustamenteconseguirá su teléfono con tal que:A. pida un reembolso de la <strong>com</strong>pañíahoy día.B. <strong>com</strong>pre minutos ilimitados.C. active una línea de servicioadicional.D. envíe un correo electrónico a ladirectora.26


Spanish (28) Practice TestLea atentamente el pasaje que sigue;luego conteste las dos preguntas acontinuación.Se invita a aquellos miembros de laasociación SML que lleven más de tresaños de afiliación a enviar propuestasal Consejo Editorial del PSML, sobrepublicaciones poco conocidas y queatraigan la atención de un amplio númerode lectores. Especialmente se buscamaterial de archivo de cualquier períodoy en cualquier idioma que no exceda las9.000 palabras. Además, si el documentono está escrito originalmente en inglés y siel proponente desea traducirlo o elegir altraductor de su preferencia (quien tambiéndebe ser miembro del SML), la propuestadebe incluir una muestra de 1.000 palabrasde la traducción; de no ser así, el traductorserá elegido por los miembros del ConsejoEditorial. El Consejo Editorial será elencargado de aceptar o rechazar laspropuestas.40. Es más probable que se dirija este pasaje:A. al Consejo Editorial de la asociaciónSML.B. a los miembros nuevos de laasociación SML.C. a cualquier miembro que deseehacer una propuesta.D. a los miembros establecidos en laasociación.39. ¿Cuál de los siguientes documentos debeentregarse al pasar la primera etapa de estepasaje?A. una copia del documentoB. una muestra de la posible traducciónC. un resumen del documentoD. un permiso del autor para publicarlo27


Spanish (28) Practice TestLea atentamente el pasaje que sigue;luego conteste las dos preguntas acontinuación.El Colegio Notarial• busca profesores con experienciadestacada <strong>com</strong>o notarios/as en laelaboración de los ÍndicesIn<strong>for</strong>matizados y• personas licenciadas que deseenadquirir experiencia en estecampo41. Se puede resumir que el Colegio Notarial:A. licencia a personas que tienenexperiencia con los Índices.B. busca gerentes que deseen dirigir unequipo de elaboración.C. necesita personas que puedanenseñar in<strong>for</strong>mática a otroscandidatos.D. provee instrucción a personas quedeseen elaborar Índices.El candidato• debe estar muy familiarizado/acon los documentos notariales yaquellos contenidos relevantespara el Índice In<strong>for</strong>matizado.• pasará a <strong>for</strong>mar parte de unequipo de personas especializadasen la elaboración de los Índices.Salario depende de la experiencia delcandidato42. Es más probable que la audiencia para estepasaje sea personas que:A. se han graduado recientemente delColegio Notarial.B. consideran cambiar sus puestos enin<strong>for</strong>mática.C. están altamente calificadas enin<strong>for</strong>mática.D. han trabajado para el ColegioNotarial antes.28


Spanish (28) Practice Test43. Lea atentamente el pasaje que sigue;luego conteste la pregunta acontinuación.La vida cotidiana del pueblo del estado deCali<strong>for</strong>nia incluye el frecuente fenómenode los temblores y las sacudidas que losa<strong>com</strong>pañan. En Cali<strong>for</strong>nia, los tembloresestán vinculados con las fallas geológicas.Los terremotos ocurren cerca de las fallasporque la corteza de la Tierra está <strong>for</strong>madapor placas independientes que estánseparadas por sistemas de fallas. Estasplacas tectónicas están siempre enmovimiento, pero estos movimientos sonnormalmente lentos e imperceptibles.Cuando una de las placas se muevebruscamente contra otra se registra unterremoto. La ciudad de San Francisco seencuentra cerca de un sistema de fallasque demarca el borde oeste de la placanorteamericana. Aunque la litósfera estásujeta a movimientos regulares cerca delas fallas, a<strong>for</strong>tunadamente muy pocosllegan a un nivel de terremoto destructivo.Según el pasaje, ¿cuál de las siguientesopciones es la razón por la cual la mayoríade los seísmos en Cali<strong>for</strong>nia no sondestructivos?A. La mayoría de los terremotosocurren a gran profundidad y poreso las vibraciones no causantemblores perceptibles en lasuperficie.B. La mayoría de los seísmos ocurrenen lugares deshabitados donde eldaño es mínimo.C. La mayoría de los movimientosde las placas tectónicas son lentosy constantes y los temblores nopueden percibirse.D. La mayoría de los movimientosde las placas tectónicas no estánasociados con las fallas geológicas.29


Spanish (28) Practice TestLea atentamente el pasaje que sigue;luego conteste las dos preguntas acontinuación.Ser vegetariano significa no <strong>com</strong>er ningúnproducto animal, ni pescado ni aves. Haymuchos estudios que demuestran losbeneficios para la salud de una dietavegetariana en vez de una de carne.Algunos se preocupan de que una dietavegetariana no pueda cumplir con losrequisitos nutritivos del cuerpo humano.Por ejemplo, los que no <strong>com</strong>en ni carne,ni huevos ni productos lácteos puedendesarrollar un déficit de vitamina B12, queabunda en la carne. La realidad es que esfácil mantener una dieta balanceada quegarantice una cantidad suficiente deproteínas con productos y raíces <strong>com</strong>o elmiso y el tempeh que crecen en la tierra.45. ¿Cuál de las siguientes oraciones describemejor la intención del autor de estepasaje?A. exponer los beneficios de unrégimen vegetarianoB. aclarar ideas equivocadas sobredietas vegetarianasC. resumir las razones por las cuales sedebe tomar vitaminasD. persuadir al público de que deje surégimen de carne44. Según este pasaje, una persona que sigueun régimen vegetariano:A. debe eliminar productos lácteos.B. tiene que tomar un suplementovitamínico.C. necesita proteínas <strong>com</strong>plejas.D. puede estar sana por <strong>com</strong>er verdurasy raíces.30


Spanish (28) Practice TestLea atentamente el pasaje que sigue; luego conteste las dos preguntas a continuación.Buenos Aires, una ciudad metropolitana de influencia europea, se abre a orillas del Río de La Plata.Es una gran selva urbana con avenidas tan importantes <strong>com</strong>o la calle Santa Fe y la Avenida Corrientes,la calle que nunca duerme. En este lugar se hace una gira gastronómica mundial, especialmente conrestaurantes italianos que ofrecen platos típicos y representativos de la Argentina. También hay lugares<strong>com</strong>o los cafés de la Plaza Dorrego y los restaurantes-bodegones donde se <strong>com</strong>e el mejor asado y sedisfruta del mejor vino en copas distintas, costumbre muy italiana entre los porteños. La capital es uncentro financiero y cultural de vanguardia, y a la vez, se mantiene la variedad de edificios en un ambientehistórico. En cuanto al diseño de su arquitectura, los amantes del urbanismo la categorizan <strong>com</strong>o unícono de referencia internacional. La capital es además escenario de museos bien conocidos <strong>com</strong>o elMuseo de Arte Latinoamericano. Allí hay obras significativas de Joán Miró y Frida Kahlo. Igualmentela capital ofrece museos especializados <strong>com</strong>o el Museo de Eva Perón y el Museo del Cine.46. ¿Cuál de los siguientes adjetivos reflejamejor el tono del autor en este pasaje?A. críticoB. preventivoC. suplicanteD. tentador47. Según el pasaje, ícono de referenciainternacional implica que Buenos Aires:A. es una ciudad cosmopolita querecibe al extranjero y lo incita adivertirse allí con los porteños.B. tiene un estilo arquitectónico querefleja las influencias tradicionalesy, a la vez, es precursor de locontemporáneo.C. se <strong>com</strong>para a otras grandes ciudadesmundiales donde hay influenciascontemporáneas, culturales yartísticas similares.D. ofrece una variedad de cocinasmundiales preparadas al estiloporteño.31


Spanish (28) Practice TestLea atentamente el pasaje que sigue; luego conteste las dos preguntas a continuación.¿Por qué la sal es un problema? El cloruro de sodio, o más <strong>com</strong>únmente llamado sal, retiene el agua enel cuerpo, lo que hace que las piernas se hinchen, que los pulmones se llenen de fluidos y que el corazónfuncione con más intensidad. Aunque le parezca difícil, con unos pocos cambios, se puede trans<strong>for</strong>marun régimen con alto contenido de sal a uno que tenga bajo contenido de sal y que además sea sabroso.De pronto, se dará cuenta que la reducción de <strong>com</strong>idas con alto contenido de sal podrá mejorarle la saludy estimularle las papilas gustativas.Para controlar la ingestión de sal, debe seguir estas re<strong>com</strong>endaciones:• Escoja <strong>com</strong>idas <strong>com</strong>o las carnes frescas, verduras y frutas de temporada y que tengan naturalmente unbajo contenido de sal.• Cocine sus <strong>com</strong>idas favoritas con poca sal o sin sal adicional.• Use una mezcla de especias con bajo contenido de sal que agregue sabor y sazón.• Disminuya el consumo de los alimentos procesados que incluyen las <strong>com</strong>idas preparadas ocongeladas "con sal agregada" en el supermercado o en los almacenes.48. Es más probable que se dirija estepasaje a:A. los fabricantes de <strong>com</strong>ida procesada.B. los entrenadores.C. los intersados en la salud.D. los cocineros.49. Según este pasaje, el punto de vista delautor es que:A. el adoptar una dieta de altocontenido de sal no será fácil paramuchas personas.B. los sabores de las <strong>com</strong>idas cambianpara muchas personas cuando seelimina el alto contenido de sal deuna dieta.C. las <strong>com</strong>idas de bajo contenido de salno están disponibles fácilmente entodos los sitios.D. el adquirir la costumbre de usaralimentos sustituidos con bajocontenido de sal beneficia la saluda largo plazo.32


Spanish (28) Practice TestLea atentamente el pasaje que sigue; luego conteste las dos preguntas a continuación.La vanguardista casa Lineski ha lanzado al mercado una nueva línea que hará subir las temperaturas delas pistas del mundo entero.El esquí de tonos escarlata Rayo <strong>com</strong>bina la suave flexibilidad de sus múltiples sectores anteriores yposteriores con la necesaria rigidez de su revolucionario refuerzo central. Ligero, y aun así todo terreno,el Rayo favorece el movimiento biomecánico natural del pie.Y para dar todavía más tono al esquí carmesí, Lineski ha producido la bota de slalom Alada, única en sugénero en precisión, equilibrio, y velocidad. La posibilidad de flexionar la parte anterior del pie se cuentaentre sus muchas ventajas.Y es que no hay mejor modo de saludar a la estación invernal que con un juego de esquís del color de lapasión.50. Según el pasaje, se puede inferir que elpropósito es proveer:A. una descripción literaria de ficción.B. un mensaje publicitario.C. un manual de instrucciones.D. un texto de divulgación científica.51. ¿Cómo debe interpretarse la expresión,una nueva línea que hará subir lastemperaturas de las pistas del mundoentero?A. El color vivo y la novedad haránpopular a este producto.B. El producto se podrá usar encualquier montaña.C. Las ventajas tecnológicas delproducto aumentarán su precio.D. Este producto permitirá esquiarincluso cuando la nieve se derrita.33


Spanish (28) Practice Test52. Lea atentamente el pasaje del Soneto alRey Nuestro Señor de Hernando deAcuña; luego conteste la pregunta acontinuación.Ya se acerca señor, o es ya llegadaLa edad gloriosa, en que promete el cieloUna grey, y un pastor, solo en el suelo,Por suerte a vuestros tiempos reservada.Ya tan alto principio en tal jornadaOs muestra el fin de vuestro santo zelo,Y anuncia al mundo para más consueloUn Monarca, un Imperio y una Espada.Se puede inferir que el tema primordial deeste poema es:A. la glorificación reflejada en elorgullo patriótico.B. imágenes que conectan la políticacon la naturaleza.C. énfasis en la idea de unidad bajocontrol militar.D. una visión del mundo estable yarmoniosa.34


Spanish (28) Practice Test53. Lea atentamente el pasaje que sigue; luego conteste la pregunta a continuación.El sacramento del bautismo, aparte de todo lo que significa para la iglesia, creó uno de los vínculosafectivos más fuertes que existen en la cultura de América Latina: el <strong>com</strong>padrazgo. El <strong>com</strong>padrazgo esuna institución en el pueblo que parte con la elección de los <strong>com</strong>padres, la que se hace algunas veces entreparientes, <strong>com</strong>o para re<strong>for</strong>zar lazos familiares, y otras veces entre los amigos. Son parientes que tienenun papel muy importante en la vida familiar de los hispanos y por eso están encargados de velar por laeducación de los hijos en el lamentable caso del fallecimiento prematuro. Las personas que aceptanesa enorme responsabilidad lo hacen porque les une un gran afecto a los padres del bebé a quien van abautizar. Hay diferentes tipos de <strong>com</strong>padrazgo: Compadre/Comadre de boquilla y Compadre/Comadrede barriga. Éste <strong>com</strong>ienza con el embarazo, es decir se le ha entregado el niño antes de su nacimiento ylleva obligaciones. Aquél es cuando dos personas acuerdan llamarse <strong>com</strong>padres sin <strong>com</strong>promiso mayor.Según el pasaje, ¿cuál de las siguientesaseveraciones explica mejor laimportancia fundamental del<strong>com</strong>padrazgo?A. Crea una afiliación familiar quehonra a la persona que cuida a losniños.B. Demuestra la unidad entre familias ala que las personas dan prioridad.C. Asegura a los hijos que sus padresles han proporcionado lo necesariopara la vida.D. Destaca la importancia de conectarlazos familiares con la educación yel cuidado de los hijos.35


Spanish (28) Practice Test54. Comprehensible-input activities in athird-year Spanish language class wouldtypically include activities in whichstudents interpret second-languagein<strong>for</strong>mation that is:A. slightly above their currentknowledge level.B. at a knowledge level equal to theircurrent second-year understanding.C. slightly below their currentknowledge level.D. at a first-year knowledge level thatthey can easily understand.56. A child learning English as her firstlanguage produces the word drived toindicate the past tense of drive. Thechild's use of the word drived most likelyresults from the child's:A. out-of-context imitation ofoverheard speech.B. misunderstanding of the past tense.C. overgeneralization of a previouslylearned rule.D. imaginative exploration of language.55. Stephen Krashen hypothesized thatsecond-language acquisition can beinhibited by:A. proactive interference of the primarylanguage.B. overextension of single words tocover concepts.C. affective filtering during<strong>com</strong>munication.D. limitations imposed by fixeddevelopmental sequences.57. Which of the following goals andstrategies <strong>for</strong> second-language teachingwould be the best way to take advantageof similarities between first- and secondlanguageacquisition?A. encouraging the development ofan extensive vocabulary throughwriting and dictionary usageB. developing native-likepronunciation and grammaticalaccuracy through drillsC. encouraging <strong>com</strong>munication ofmeaning through holistic learningmethodsD. developing a sophisticatedunderstanding of literature throughintensive reading36


Spanish (28) Practice Test58. Providing direct instruction in thegrammar of a language and makingcorrections to a learner's mistakes aremost likely to be helpful to:A. young children learning a firstlanguage.B. adults in the process of learning anew language.C. young children being raised to speaktwo languages.D. adults learning to speak a differentdialect of their first language.60. In second-language acquisition,intermediate proficiency is characterizedby which of the following properties?A. dependence on props <strong>for</strong><strong>com</strong>municatingB. ability to negotiate meaning inconversation and other dailyinteractionsC. cautious use of vocabulary limitedto concrete objects and simplephrasesD. reading and speaking skills similarto those of a native speaker59. Which of the following best characterizesthe <strong>com</strong>prehension stage of secondlanguagedevelopment?A. production of partially correctstructures when speakingB. inconsistent use of correctgrammatical constructionsC. speaking in simplified but accuratesentences to express ideasD. interpretation of the language withlittle or no production61. A high school Spanish teacher is teachinga course that includes students with manydifferent backgrounds, experiences, andlearning styles. Which of the followingtypes of assignments would bestac<strong>com</strong>modate the diverse needs of thestudents?A. exercises from the students'textbookB. cooperative and collaborativeassignmentsC. individual Internet and <strong>com</strong>puterworkD. presentational projects prepared athome37


Spanish (28) Practice Test62. In which of the following ways coulda Spanish teacher apply conceptsof language acquisition and literacydevelopment to help students understanda newspaper article in Spanish?A. conducting pre-reading activitieswith students be<strong>for</strong>e they read thearticleB. providing students with an Englishtranslation of the articleC. asking students questions aboutsignificant details after they havefinished reading the articleD. directing students to take turnsreading the article aloud to a partner63. Which of the following sentences isgrammatically correct?A. Manolo estaba eufórico porquese había elegido a este puestomunicipal.B. Manolo estaba eufórico por habersido elegido para este puestomunicipal.C. Manolo estaba eufórico a causade haber elegido a este puestomunicipal.D. Manolo estaba eufórico por haberseelegido a este puesto muncipal.64. Which of the following sentencescorrectly <strong>com</strong>bines the two sentencesbelow while preserving the meaning ofthe original sentences?Los estudiantes no han contestado todaslas preguntas. No hay por qué darles unabuena nota.A. Los estudiantes no han contestadotodo, <strong>com</strong>o consecuencia no sacaránbuenas notas.B. Los estudiantes han contestado todo,<strong>com</strong>o resultado no tendrán buenasnotas.C. Como consecuencia de no habercontestado todo, los estudiantesobtendrán buenas notas.D. Hay razón para darles buenas notasporque contestaron todo.65. In Spanish, some nouns can function asadverbs when they are preceded by whichof the following words?A. sumamenteB. muyC. conD. más38


Spanish (28) Practice Test66. Which of the following changes correctsan error in the sentence below?Me di cuenta que no he apagado elteléfono celular cuando estaba paraempezar el concierto a las dos de la tarde.A. Me di cuenta → Me daba cuentaB. he apagado → había apagadoC. estaba → estuvoD. de → por67. Which of the following options is a moreaccurate way to restate the question in thedialogue below?Camarero: Tenemos todo tipo demariscos: almejas, camarones, cangregosy langostas.Cliente: ¿En cuál método estáncocinados?A. ¿Cómo los prepara?B. ¿Cuál es la manera en la que se losprepara?C. ¿Cómo se los prepara?D. ¿Podría hacerme el favor de explicarcómo las prepara?68. Which of the following options correctsthe sentence below?El perro se escapó de la puerta después dela niña lo dejó abierto.A. El perro se escapaba por la puertadespués de que la niña la dejabaabierta.B. El perro se escapó de la puertacuando la niña la había abierto.C. El perro se escapaba de la puertacuando la niña la ha abierto.D. El perro se escapó por la puertadespués de que la niña la dejóabierta.69. Which of the following sets ofdemonstrative pronouns correctly<strong>com</strong>pletes the sentence below?Bolívar y San Martín son dos héroes de laindependencia sudamericana. ________era argentino, ________, venezolano.A. Éste / éseB. Éste / aquélC. Eso / aquelloD. Ése / aquél39


Spanish (28) Practice Test70. Which of the following sets of wordscorrectly <strong>com</strong>pletes the dialogue below?Dependiente: Buenas tardes, señor. ¿____________________?Cliente: Me gustaría ver las camisas queestán rebajadas, señora. ¿Las tiene de linoy de algodón?Dependiente: Sí, señor. Las tenemos.________ en el probador que está detrásdel cajero.A. ¿En qué puedo servirle? /PruébatelasB. ¿Qué buscas Ud.? / Ud. puedeponérselasC. ¿Hay algo que busque? /PóngaselasD. ¿En qué puedo servirle? / Ud.puede probárselas72. Which of the of the following sentencescontains an example of the sound [ŋ] as inthe English word singing?A. Están nerviosos.B. Son famosas.C. Han ganado.D. Prefieren mudarse.73. In which of the following words is thefirst letter pronounced in some Spanishspeakingareas like /h/ in the English wordhockey?A. hambreB. jirafaC. gordoD. hombro71. Which of the following options correctlytrans<strong>for</strong>ms the sentence below into anaffirmative sentence?Nunca veía a nadie en ese teatro por latarde.A. Veía a todos en ese teatro por latarde.B. Siempre veía a alguien en ese teatropor la tarde.C. Tampoco veía a todos en este teatropor la tarde.D. Veía a alguien en ese teatro por latarde.74. Which of the following sets of Spanishwords best <strong>com</strong>pletes the sentence belowwith a meaning equivalent to the Englishwords underlined in the first sentence?Barbara decided to study law with thatgroup of distinguished faculty.Bárbara decidió estudiar ________ conese grupo de distinguido ________.A. derecho / profesoradoB. leyes / facultadC. ley / facultadesD. derechos / profesorados40


Spanish (28) Practice Test75. Which of the following responsesincludes both the best verb to <strong>com</strong>pletethe sentence below and a correct rationale<strong>for</strong> its use?Cuando la tormenta empezó, de repenteel cielo ________ muy oscuro.A. estaba—indicates a changeablephysical, mental or emotional stateB. se volvió—means a sudden changein the physical environmentC. se puso—shows changes inphysical, mental, or emotional statesD. era—indicates a physicaldescription of the environment76. A literary character who is referred to asquixotic is most likely someone who is:A. foolish.B. practical.C. idealistic.78. In general, use of usted would be mostappropriate in which of the followingsituations?A. a mother talking to her grown childB. a sister talking to her older brotherC. a girl talking to her boyfriendD. a child talking to an elderly woman79. In U.S. stores, customers may be greetedin<strong>for</strong>mally when transacting business.Which of the following statements wouldmost likely be used by a vendor when acustomer enters a store or shop in LatinAmerica?A. ¡Hola! Todo está en oferta.B. Buenos días. ¿En qué puedoservirle?C. Buenas tardes. Permítame el honorde servirle en esta tienda.D. Pasa, por favor. ¿Qué buscas?D. driven.77. The word peso can be tracedetymologically to:A. monetary units.B. weight of currency.C. shape and <strong>for</strong>m of money.D. currency exchange.41


Spanish (28) Practice Test80. The presidency of Augusto Pinochetin Chile (1973–1988) is most closelyassociated with which of the followingdevelopments?A. the creation of a regional tradenetworkB. the rise of ethnic and religiousconflictC. the globalization of the regionaleconomyD. the suppression of democraticliberties81. After gaining independence, SouthAmerican nations experienced sloweconomic development <strong>for</strong> half a centuryprimarily because of:A. climatic variations due to naturaldisasters.B. poor trade connections withcountries other than Spain.C. a lack of significant naturalresources.D. internal and regional conflicts overleadership and territory.82. Which of the following best describes asignificant contribution of Benito Juárezto the development of Mexico?A. As president, he led a movementthat overthrew <strong>for</strong>eign rule andsought to create a democraticfederal republic in Mexico.B. He led a revolution that ended a27-year dictatorship and restoredcivil liberties.C. He <strong>com</strong>manded the militaryresistance of the Maya against eliteoppression of indigenous peoples inthe Yucatán region of Mexico.D. Under his dictatorship, the Mexicaneconomy expanded and the standardof living improved.83. In which of the following ways has theAndes Mountain Range most affectedagricultural production in Colombia?A. The mountain slopes provide anextensive region <strong>for</strong> growing coffeeas a major export crop.B. Steep and rugged terrain makesmechanized agriculture impossibleand limits production to smallmarketand subsistence crops.C. Terraced hillsides and abundantstreams enable production of wheat<strong>for</strong> the export market.D. The mountains' stony and thin soilsare suitable primarily <strong>for</strong> grazingsheep, goats, and llamas thatprovide wool <strong>for</strong> textile industries.42


Spanish (28) Practice Test84. Spain's democratic system of governmentin the early twenty-first century isprimarily a result of:A. a military coup that overthrew themonarchy in the early twentiethcentury.B. ef<strong>for</strong>ts of Francisco Franco tomodernize and develop the countryfollowing World War II.C. constitutional changes initiated byKing Juan Carlos after the death ofFrancisco Franco in the 1970s.D. a coalition <strong>for</strong>med between theCommunist Party and the SocialistParty in the late twentieth century.85. In seventeenth- and eighteenth-centuryNorth America, Spanish colonialsettlement differed significantly fromBritish colonial settlement because theSpanish were more likely to:A. incorporate indigenous people intotheir society through religiousconversion and employment as alabor <strong>for</strong>ce.B. negotiate with indigenous groups assovereign nations <strong>for</strong> the purchaseof territory and settlement rights.86. Which of the following aspects of Peru'sarchitectural heritage is preserved atMachu Picchu?A. elaborate Spanish colonialcathedrals and public buildingsB. the small stone and timber structurestypical of indigenous village lifeC. stonework and terracing of an Incapolitical and ceremonial centerD. the main house and outbuildings ofa typical Spanish colonial hacienda87. A study of the literary contributions ofRubén Darío would be most useful <strong>for</strong>developing understanding of which of thefollowing aspects of literature?A. the ways in which literature reflectsnational beliefs and identityB. the character of early twentiethcenturymodernist literatureC. the influence of folklore on literarytraditionsD. the themes of late nineteenthcenturyromantic literatureC. limit interactions with indigenouspeople to trade relations andmilitary alliances.D. provide opportunities <strong>for</strong> indigenousleaders to receive education andassume roles in the administrationof territorial government affairs.43


Spanish (28) Practice Test88. In which of the following everydaycultural practices in the United Statesis Hispanic influence most prevalent?A. popular musicB. detective novelsC. technologyD. television <strong>com</strong>mercials89. The epic poem El gaucho Martín Fierroby José Hernández portrays the Argentinegaucho as a character whose solitary lifein the open taught him self-reliance,courage, indifference to hardship, and loveof the land. The popularity of this poemduring Argentina's early national periodcan best be attributed to its:A. celebration of Argentina'sindigenous population in vividand sensitive descriptions.B. affirmation of the value of progressin Argentina's agriculturalproduction.C. embodiment of Argentine characterand the essence of the national soul.D. descriptions of political conflicts inthe Argentine wilderness.90. Which of the following best describes amajor aim of the novelists, essayists, andpoets of Spain's Generation of 1898?A. to expand the meaning of culturethrough a blending of myth, magic,and realismB. to establish a synthesis of religiousand secular elements in SpanishcultureC. to explore the artistic value oflanguage through experiments withsymbolic representationD. to promote public debate on thenature of Spain, its people, and itsgovernment91. Which of the following is a distinctivecharacteristic of twentieth-century LatinAmerican literature?A. the use of elaborate, sensory andrhythmic language as an expressionof artistic mediumB. themes that are focused on culturalrelativity and regional identityC. the use of narrative strategies that<strong>com</strong>bine elements of fantasy andmyth with authentic storylinesD. emphasis on historicalconsciousness and reflection44


Spanish (28) Practice Test92. Use the in<strong>for</strong>mation below to answerthe question that follows.• Rudolfo Anaya, Bless Me, Ultima• Sandra Cisneros, The House onMango Street• Richard Rodriguez, Hunger ofMemoryOne major theme <strong>com</strong>mon to works ofChicano literature in the late twentiethcentury, such as those listed in the boxabove, is a focus on the:A. sense of loss and cultural dislocationexperienced by immigrants in a newcountry.B. perspectives and consequences ofpolitical struggles in Latin Americancountries.C. challenges of negotiating a Latinocultural identity in a non-Hispanicsociety.D. social tensions of class, race, andethnicity in contemporary LatinAmerican societies.93. Which line in the chart below correctlymatches a style of music with the LatinAmerican country in which it originated?LineStyle ofMusicLatin AmericanCountry1 salsa Mexico2 corridos Colombia3 samba Guatemala4 tango ArgentinaA. Line 1B. Line 2C. Line 3D. Line 494. In which of the following ways dotraditional Spanish family practicestend to differ most from the practicesof families in the United States?A. Spanish youths are more likely tolive with their parents until theymarry.B. Individuals in Spain are less likelyto engage in leisure activities thatinclude other family members.C. Spanish parents are less likely tosupport their children throughcollege.D. Young children in Spain are morelikely to remain home when olderfamily members dine out.45


Spanish (28) Practice Test95. In Mexico, addressing a person by the titleLicenciado is most appropriate when theindividual:A. is a medical doctor.B. has served in the military.C. is a civil servant.D. has a university degree.98. According to traditional namingconventions in Spanish-speakingcountries, the children of José PérezMontilla and Alicia Abascal Blanco wouldhave which of the following surnames?A. Pérez BlancoB. Montilla AbascalC. Pérez AbascalD. Montilla Blanco96. Cultural practices in which of thefollowing Latin American countries mostreflect the influence of African cultures?A. BoliviaB. ChileC. Dominican RepublicD. Argentina99. In Spanish-speaking countries, when awoman marries and chooses to add herhusband's last name, it is preceded by:A. a.B. y.C. de.D. con.97. During the late twentieth century, PuertoRico became one of the world's leadingmanufacturers of pharmaceuticals.Which of the following factors wasmost significant in the development ofPuerto Rico's pharmaceutical industry?A. the mineral and biological resourcesof the island's Cordillera Central andtropical <strong>for</strong>estsB. the island's political and economicrelationship with the United States100. An Argentine restaurant in the UnitedStates is more likely than a Mexicanrestaurant to focus its menu on the use of:A. potatoes.B. rice and beans.C. beef.D. hot peppers.C. the presence of energy resourcesin the island's coastal plains andcoastal ocean territoryD. the island's large and highly skilledlabor pool46


Spanish (28) Practice TestDIRECTIONS FOR SECTION THREE: WRITTEN EXPRESSIONThis section of the test consists of a writing assignment to which you must respond in the target language. Theassignment appears on the following page. A list of suggestions is provided to help direct your writing <strong>for</strong> theassignment. It is not necessary that you cover every point on the list, nor are you limited in your response to thosepoints indicated. You are, however, required to write about the general assignment that you are given, and partof your score will be based on the degree to which you elaborate on the assignment by addressing either thesuggested points or points of your choosing.You may use any blank space on the unlined pages following the writing assignment to make notes, write anoutline, or otherwise prepare your response.Your response will be evaluated based on the following criteria.• PURPOSE: the extent to which the objective of the assignment is achieved and the response isappropriate <strong>for</strong> the intended audience• COHERENCE: organization and clarity of ideas• CONTENT: development of ideas and relevance of supporting details• GRAMMAR: accuracy of grammatical <strong>for</strong>ms and syntax• VOCABULARY: <strong>com</strong>mand of vocabulary and idiomatic expressions• MECHANICS: accuracy of spelling, diacritical marks, and punctuationBe sure to write about the assigned topic and use MULTIPLE PARAGRAPHS. Please write legibly. You maynot use any reference materials during the test. This should be your original work, written in your own words,and not copied or paraphrased from some other work. Remember to review what you have written and make anychanges you think will improve your written response.Write or print your response in the space provided following the assignment.47


Spanish (28) Practice TestOPEN-RESPONSE ITEM: WRITTEN EXPRESSIONImagine that your school is hosting a teacher from a Spanish-speaking country to spend time in your<strong>com</strong>munity. The teacher has requested in<strong>for</strong>mation on the culture of the area in which you live. Writingin Spanish, prepare a letter to the teacher describing the culture of the area in which you live (e.g., sports,entertainment, museums, etc.) and recounting an experience you or someone you know once hadinvolving a local cultural activity.You may wish to include, but are not limited to, the following topics:• cultural activities that are especially popular where you live;• why you think these cultural activities are especially popular; and• cultural activities you think may be of interest to the teacher.48


Spanish (28) Practice TestOPEN-RESPONSE SHEET: WRITTEN EXPRESSION______________________________________________________________________________________________________________________________________________________________________________________________________________________________________________________________________________________________________________________________________________________________________________________________________________________________________________________________________________________________________________________________________________________________________________________________________________________________________________________________________________________________________________________________________________________________________________________________________________________________________________________________________________________________________________________________________________________________________________________________________________________________________________________________________________________________________________________________________________________________________________________________________________________________________________________________________________________________________________________________________________________________________________________________________________________________________________________________________________________________________________________________________________________________________________________________________________________________________________________________________________________________________________________________________________________________________________________________________________________________________________________________________________________________________________________________________________________________________________________________________________________________________________________________________________________________________________________________________________________________________________________________________________________________________________________________________________________________________________________________________________________________________________________________________________________________________________________________49


Spanish (28) Practice TestOPEN-RESPONSE SHEET: WRITTEN EXPRESSION______________________________________________________________________________________________________________________________________________________________________________________________________________________________________________________________________________________________________________________________________________________________________________________________________________________________________________________________________________________________________________________________________________________________________________________________________________________________________________________________________________________________________________________________________________________________________________________________________________________________________________________________________________________________________________________________________________________________________________________________________________________________________________________________________________________________________________________________________________________________________________________________________________________________________________________________________________________________________________________________________________________________________________________________________________________________________________________________________________________________________________________________________________________________________________________________________________________________________________________________________________________________________________________________________________________________________________________________________________________________________________________________________________________________________________________________________________________________________________________________________________________________________________________________________________________________________________________________________________________________________________________________________________________________________________________________________________________________________________________________________________________________________________________________________________________________________________________________50


Spanish (28) Practice TestDIRECTIONS FOR SECTION FOUR: ORAL EXPRESSION(You will see in the test booklet:)For this section of the test, you will speak on tape in response to an assignment presented in your test booklet.You will read the assignment in your test booklet and have one minute to consider your response, then twominutes to speak. You will be provided with a list of suggestions to help direct your response. It is not necessarythat you address every point in the list, nor are you limited to the points mentioned. However, what you say mustbe relevant to the situation with which you are presented, and part of your score will be based on the degree towhich you elaborate on the topic by addressing either the suggested points or points of your choosing. You mayNOT write in this test booklet. You will be scored only on the response you provide on the audiotape afterthe announcer instructs you to begin speaking.Your response will be evaluated based on the following criteria.• CONTENT: the adequacy of the in<strong>for</strong>mation presented, including the number of ideas, the developmentof ideas, and the depth and specificity of supporting details• COHERENCE: the organization and clarity of the ideas <strong>com</strong>municated• GRAMMAR: the accuracy of grammatical and syntactic constructions (e.g., conjugation of verbs,appropriate use of tenses, subject/pronoun and article/noun agreement)• VOCABULARY: the <strong>com</strong>mand of vocabulary and idiomatic expressions and appropriateness of wordchoice• FLUENCY: the ability to maintain a flow of speech, without hesitations or pauses that affect the overall<strong>com</strong>prehensibility of the response• PRONUNCIATION: the <strong>com</strong>prehensibility of articulation and the appropriateness of stressOnce recording has begun, it cannot be stopped, nor can you re-record your response.When you have finished reading these directions, please look up. You will receive further instructions from yourtest administrator.Look up when you are ready to begin Section Four.STOPDO NOT GO ON UNTIL YOU ARE TOLD TO DO SO.51


Spanish (28) Practice TestOPEN-RESPONSE ITEM: ORAL EXPRESSION(You will see in the test booklet and hear on the recording:)Read the instructions below. You will have one minute to study these instructions. Then, when youare told to do so, begin to respond in Spanish. You may take up to two minutes to <strong>com</strong>plete yourresponse. You will be told when the two minutes are up. You may begin studying the instructionsnow. You will be told when to begin speaking.(You will see in the test booklet:)Imagine that you are talking with a Spanish-speaking friend. Your friend is considering beginning a newhobby or pastime. Speaking in Spanish, discuss hobbies and pastimes with your friend and offer youradvice. You may wish to include, but are not limited to, the following:• a description of hobbies or pastimes you enjoy;• your advice to your friend about a hobby or pastime you think your friend would enjoy; and• why you think this hobby or pastime is particularly appropriate <strong>for</strong> your friend.(You will have one minute to study the instructions above. Then you will have two minutes to respond onthe audiotape.)52


Spanish (28) Practice TestPRACTICE TEST RESULTS53


Spanish (28) Practice TestPRACTICE TEST RESULTS OVERVIEWThe practice test provides valuable in<strong>for</strong>mation regarding your preparedness <strong>for</strong> the <strong>MTEL</strong> Spanish (28) test. Inthis section, you will find in<strong>for</strong>mation and tools to help you determine your preparedness on the various sectionsof the test.Multiple-Choice QuestionsA Multiple-Choice Question Answer Key Worksheet is provided to assist you in evaluating your multiple-choiceresponses. The worksheet contains five columns. The first column indicates the multiple-choice questionnumber, the second column indicates the objective to which the test question was written, and the third columnindicates the correct response. The remaining columns are <strong>for</strong> your use in calculating the number of multiplechoicequestions you answered correctly or incorrectly.An Evaluation Chart <strong>for</strong> the multiple-choice questions is also provided to help you assess which content coveredby the test objectives may require additional study.Open-Response ItemsEvaluation In<strong>for</strong>mation, Sample Responses and Analyses, as well as a Scoring Rubric are provided <strong>for</strong> theseitems. You may wish to refer to this in<strong>for</strong>mation when evaluating your practice test responses.Total TestPractice Test Score Calculation in<strong>for</strong>mation is provided to help you estimate your score on the practice test.Although you cannot use this practice test to precisely predict how you might score on an official <strong>MTEL</strong> Spanish(28) test, you may be able to determine your degree of readiness to take an <strong>MTEL</strong> test at an operationaladministration. No passing score has been determined <strong>for</strong> the practice test.54


Spanish (28) Practice TestMULTIPLE-CHOICE QUESTIONANSWER KEY WORKSHEETQuestion Objective Correct Your ResponseNumber Number Response Correct? Incorrect?1 0001 A2 0001 C3 0002 C4 0001 C5 0003 B6 0001 D7 0002 A8 0001 A9 0001 A10 0001 D11 0002 C12 0001 B13 0002 C14 0001 C15 0002 C16 0002 A17 0002 B18 0002 C19 0003 D20 0002 D21 0003 C22 0003 D23 0003 B24 0003 C25 0003 B26 0003 C27 0004 C28 0006 A29 0004 C30 0004 B31 0005 C32 0004 D33 0006 C34 0004 C55


Spanish (28) Practice TestMULTIPLE-CHOICE QUESTIONANSWER KEY WORKSHEET (continued)Question Objective Correct Your ResponseNumber Number Response Correct? Incorrect?35 0004 A36 0006 B37 0004 B38 0005 C39 0004 B40 0005 D41 0004 D42 0005 C43 0005 C44 0005 D45 0006 B46 0005 D47 0006 B48 0005 C49 0006 D50 0005 B51 0006 A52 0006 A53 0006 D54 0007 A55 0007 C56 0007 C57 0007 C58 0007 B59 0007 D60 0007 B61 0007 B62 0007 A63 0008 B64 0008 A65 0008 C66 0008 B67 0008 C68 0008 D56


Spanish (28) Practice TestMULTIPLE-CHOICE QUESTIONANSWER KEY WORKSHEET (continued)Question Objective Correct Your ResponseNumber Number Response Correct? Incorrect?69 0008 B70 0008 D71 0008 B72 0009 C73 0009 B74 0009 A75 0009 C76 0009 C77 0009 B78 0009 D79 0009 B80 0010 D81 0010 D82 0010 A83 0010 A84 0010 C85 0010 A86 0010 C87 0011 B88 0011 A89 0011 C90 0011 D91 0011 C92 0011 C93 0011 D94 0012 A95 0012 D96 0012 C97 0012 B98 0012 C99 0012 C100 0012 CCount the number of multiple-choice questions you answered correctly:__________ of 100 multiple-choice questions57


Spanish (28) Practice TestMULTIPLE-CHOICE QUESTIONPRACTICE TEST EVALUATION CHARTIn the evaluation chart that follows, the multiple-choice questions are arranged in numerical order and by testobjective. Check your responses against the correct responses provided to determine how many questions withineach objective you answered correctly.Subarea I: Listening ComprehensionObjective 0001: Derive in<strong>for</strong>mation from a variety of culturally authentic oral messages(e.g., telephone message, radio broadcast, speech, lecture, public addressannouncement, conversation) in the target language.1A_____ 2C_____ 4C_____ 6D_____ 8A_____ 9A_____ 10D_____ 12B_____14C__________/9Objective 0002: Apply skills of inference to a variety of culturally authentic oral messages(e.g., telephone message, radio broadcast, speech, lecture, public addressannouncement, conversation) in the target language.3C_____ 7A_____ 11C_____ 13C_____ 15C_____ 16A_____ 17B_____ 18C_____20D__________/9Objective 0003: Analyze a variety of culturally authentic oral messages (e.g., telephone message,radio broadcast, speech, lecture, public address announcement, conversation)in the target language.5B_____ 19D_____ 21C_____ 22D_____ 23B_____ 24C_____ 25B_____ 26C__________/8Subarea I (Objectives 0001–0003) Total _____/2658


Spanish (28) Practice TestMULTIPLE-CHOICE QUESTIONPRACTICE TEST EVALUATION CHART (continued)Subarea II: Reading ComprehensionObjective 0004: Understand the literal content of a variety of authentic materials(e.g., brochure, Web site, newspaper article, letter, literary or philosophical work)written in the target language.27C_____ 29C_____ 30B_____ 32D_____ 34C_____ 35A_____ 37B_____ 39B_____41D__________/9Objective 0005: Apply skills of inference to a variety of authentic materials (e.g., brochure, Website, newspaper article, letter, literary or philosophical work) written in the target language.31C_____ 38C_____ 40D_____ 42C_____ 43C_____ 44D_____ 46D_____ 48C_____50B__________/9Objective 0006: Analyze a variety of authentic materials (e.g., brochure, Web site, newspaperarticle, letter, literary or philosophical work) written in the target language.28A_____ 33C_____ 36B_____ 45B_____ 47B_____ 49D_____ 51A_____ 52A_____53D__________/9Subarea II (Objectives 0004–0006) Total _____/2759


Spanish (28) Practice TestMULTIPLE-CHOICE QUESTIONPRACTICE TEST EVALUATION CHART (continued)Subarea III: Linguistics and Language StructuresObjective 0007: Demonstrate an understanding of the nature of first- andsecond-language acquisition.54A_____ 55C_____ 56C_____ 57C_____ 58B_____ 59D_____ 60B_____ 61B_____62A__________/9Objective 0008: Understand and apply simple and <strong>com</strong>plex linguistic structuresof the target language.63B_____ 64A_____ 65C_____ 66B_____ 67C_____ 68D_____ 69B_____ 70D_____71B__________/9Objective 0009: Understand the similarities and differences between the target languageand English.72C_____ 73B_____ 74A_____ 75C_____ 76C_____ 77B_____ 78D_____ 79B_____ _____/8Subarea III (Objectives 0007–0009) Total _____/2660


Spanish (28) Practice TestMULTIPLE-CHOICE QUESTIONPRACTICE TEST EVALUATION CHART (continued)Subarea IV: Cultural Perspectives, Comparisons, and ConnectionsObjective 0010: Understand major developments in the history of target-language-speakingcultures and the significance of these developments.80D_____ 81D_____ 82A_____ 83A_____ 84C_____ 85A_____ 86C__________/7Objective 0011: Understand the products and perspectives of target-language-speaking cultures.87B_____ 88A_____ 89C_____ 90D_____ 91C_____ 92C_____ 93D__________/7Objective 0012: Understand the practices and perspectives of target-language-speaking cultures.94A_____ 95D_____ 96C_____ 97B_____ 98C_____ 99C_____ 100C__________/7Subarea IV (Objectives 0010–0012) Total _____/2161


Spanish (28) Practice TestOPEN-RESPONSE ITEM EVALUATION INFORMATIONHow Open-Response Items Are ScoredOpen-response items are scored through a process called focused holistic scoring. Scorers judge the overalleffectiveness of the response rather than individual aspects considered in isolation.How to Evaluate Your Practice ResponsesOn the following pages, you will find two "strong" and two "weak" sample responses. PLEASE DO NOTREVIEW THE SAMPLE RESPONSES UNTIL AFTER YOU HAVE WRITTEN YOUR OWN RESPONSE.When you do review the two "strong" and "weak" sample responses and analyses included here, please note thefollowing points: For the purposes of the practice test, responses are identified as "strong" or "weak" rather than given ascore point of 1–4. The responses identified as "strong" may contain flaws; however, these responses do demonstrate theper<strong>for</strong>mance characteristics of a "strong response." The two "strong" responses demonstrate the examinees' appropriate understanding and application of thesubject matter knowledge. However, these responses do not necessarily reflect the full range of "correctanswers" that would demonstrate an understanding of the subject matter. The "Analysis" ac<strong>com</strong>panying each "strong" and "weak" response discusses the main attributes of theresponses, but does not identify all flaws or strengths that may be present.Compare your practice responses to the Sample Responses to determine whether your responses are more similarto the strong or weak responses. Also review the Analyses on those pages and the Scoring Rubric to help youbetter understand the characteristics of strong and weak responses. This evaluation will help you identify specificproblems or weaknesses in your practice responses. Further in<strong>for</strong>mation on scoring can be found in the TestIn<strong>for</strong>mation Booklet and Faculty Guide at <strong>www</strong>.<strong>mtel</strong>.<strong>nesinc</strong>.<strong>com</strong> and at <strong>www</strong>.doe.mass.edu/<strong>mtel</strong>; select "FAQ,"then "After the Test."62


Spanish (28) Practice TestOPEN-RESPONSE ITEMSCORING RUBRICS, SAMPLE RESPONSES,AND ANALYSES63


Spanish (28) Practice TestPer<strong>for</strong>mance Characteristics:PurposeCoherenceContentGrammarVocabularyMechanicsScoring Scale:ScorePoint4321UB<strong>Massachusetts</strong> <strong>Tests</strong> <strong>for</strong> <strong>Educator</strong> <strong>Licensure</strong>®SCORING RUBRIC FOR SUBJECT TESTFOREIGN LANGUAGE ASSIGNMENTS—WRITTEN EXPRESSIONThe extent to which the objective of the assignment is achieved and the appropriateness of theresponse <strong>for</strong> the intended audience.Organization and clarity of ideas.Development of ideas and relevance of supporting details.Accuracy of grammatical <strong>for</strong>ms and syntax.Command of vocabulary and idiomatic expressions.Accuracy of spelling, diacritical marks, and punctuation.Score Point DescriptionThe "4" response reflects a thorough application and strong <strong>com</strong>mand of the per<strong>for</strong>mance characteristics <strong>for</strong> the writtenassignment.• The response thoroughly fulfills the purpose of the written assignment and is <strong>com</strong>pletely appropriate <strong>for</strong> the intended audience.• The candidate's ideas are well organized and clearly expressed.• Ideas are extensively developed and well-supported with relevant in<strong>for</strong>mation.• The response shows a <strong>com</strong>prehensive <strong>com</strong>mand of syntax and grammar, with only minor errors that do not interrupt<strong>com</strong>munication.• Vocabulary reflects a broad <strong>com</strong>mand of the language and appropriate use of idiomatic expressions.• Spelling, diacritical marks, and punctuation are mastered, with few, if any, errors.The "3" response reflects an adequate application and satisfactory <strong>com</strong>mand of the per<strong>for</strong>mance characteristics <strong>for</strong> thewritten assignment.• The response satisfactorily fulfills the purpose of the written assignment and is generally appropriate <strong>for</strong> the intended audience.• The candidate's ideas show some organization and are generally clear.• Ideas are adequately developed with some elaboration and support of specific points.• The response shows a good <strong>com</strong>mand of syntax and grammar, though some errors may cause minor interruptions in<strong>com</strong>munication.• Vocabulary and idiomatic expressions are general, but adequate <strong>for</strong> <strong>com</strong>municating a <strong>com</strong>plete message.• There are minor errors in spelling, diacritical marks, and punctuation that do not interfere with <strong>com</strong>munication.The "2" response reflects a partial application and limited <strong>com</strong>mand of the per<strong>for</strong>mance characteristics <strong>for</strong> the writtenassignment.• The response partially fulfills the purpose of the written assignment and may not be entirely appropriate <strong>for</strong> the intendedaudience.• The candidate's ideas show limited organization and are somewhat unclear.• Ideas are developed in a limited way with minimal support.• The response shows a limited <strong>com</strong>mand of syntax and grammar, with frequent errors that partially impede <strong>com</strong>munication ofideas.• Vocabulary is simple and lacks key words and expressions but is sufficient <strong>for</strong> <strong>com</strong>municating a partial message.• There are some errors in spelling, diacritical marks, and punctuation, which partially interfere with <strong>com</strong>munication.The "1" response reflects an inadequate application and a lack of <strong>com</strong>mand of the per<strong>for</strong>mance characteristics <strong>for</strong> thewritten assignment.• The response fails to fulfill the purpose of the written assignment and is inappropriate <strong>for</strong> the intended audience.• The candidate's ideas are unorganized and unclear.• Ideas are developed minimally, if at all, and lack any relevant supporting detail.• The response shows little <strong>com</strong>mand of basic elements of grammar or syntax, with numerous and frequent errors that impede<strong>com</strong>munication.• Vocabulary is not adequate <strong>for</strong> <strong>com</strong>municating a <strong>com</strong>plete message, with word usage errors and mis<strong>for</strong>mations that impede<strong>com</strong>munication.• Errors in spelling, diacritical marks, and punctuation are so numerous that they impede <strong>com</strong>munication.The response is unrelated to the assigned topic, illegible, not written in the required language, not of sufficientlength to score, or merely a repetition of the assignment.There is no response to the assignment.64


Spanish (28) Practice TestPer<strong>for</strong>mance Characteristics:ContentCoherenceGrammarVocabularyFluencyPronunciationScoring Scale:ScorePoint4321UB<strong>Massachusetts</strong> <strong>Tests</strong> <strong>for</strong> <strong>Educator</strong> <strong>Licensure</strong>®SCORING RUBRIC FOR SUBJECT TESTFOREIGN LANGUAGE ASSIGNMENTS—ORAL EXPRESSIONAdequacy of the in<strong>for</strong>mation presented, including the number of ideas, the development of ideas, and thedepth and specificity of supporting details.Organization and clarity of ideas <strong>com</strong>municated.Accuracy of grammatical and syntactic constructions (e.g., conjugation of verbs, appropriate use of tenses,subject/pronoun and article/noun agreement).Command of vocabulary and idiomatic expressions and appropriateness of word choice.Ability to maintain a flow of speech, without hesitation or pauses which affect the overall <strong>com</strong>prehensibilityof the response.Comprehensibility of articulation and the appropriateness of stress.Score Point DescriptionThe "4" response reflects a thorough application and strong <strong>com</strong>mand of the per<strong>for</strong>mance characteristics <strong>for</strong> thespeaking assignment.• The speaker's ideas are well developed and elaborated.• The candidate's ideas are logically sequenced, well connected, and consistently clear.• The response shows a <strong>com</strong>prehensive <strong>com</strong>mand of syntax and grammar, making only minor errors that do not interfere withthe <strong>com</strong>prehensibility of ideas.• The candidate demonstrates extensive <strong>com</strong>mand of appropriate vocabulary and idiomatic expressions.• The candidate maintains a strong flow of speech, with few, if any, hesitations or pauses.• The speaker's pronunciation is easily intelligible, with few, if any, errors.The "3" response reflects an adequate application and satisfactory <strong>com</strong>mand of the per<strong>for</strong>mance characteristics <strong>for</strong> thespeaking assignment.• The speaker's ideas are adequately developed, with some elaboration.• The candidate's ideas are adequately organized and are generally clear.• The response shows good <strong>com</strong>mand of syntax and grammar, though some errors may cause minor interruptions in the<strong>com</strong>prehensibility of ideas.• The candidate demonstrates a good <strong>com</strong>mand of generally appropriate vocabulary and idiomatic expressions.• The candidate generally maintains a steady flow of speech, with only occasional hesitations and pauses.• The speaker's pronunciation is generally intelligible, with only minor errors.The "2" response reflects a partial application and limited <strong>com</strong>mand of the per<strong>for</strong>mance characteristics <strong>for</strong> the speakingassignment.• The speaker's ideas show limited development and elaboration.• The candidate's ideas are only partially organized and are at times unclear.• The response shows a limited <strong>com</strong>mand of syntax and grammar, with frequent errors that partially impede the<strong>com</strong>prehensibility of ideas.• The candidate demonstrates a limited range of expression, using very simple vocabulary and lacking some key words andphrases.• The candidate does not maintain a consistent flow of speech, having frequent hesitations and pauses.• The speaker's pronunciation contains several errors and is occasionally difficult to understand.The "1" response reflects an inadequate application and a lack of <strong>com</strong>mand of the per<strong>for</strong>mance characteristics <strong>for</strong> thespeaking assignment.• The speaker's response provides few, if any, <strong>com</strong>prehensible ideas, with minimal development.• The candidate's ideas are not organized and are largely unclear.• The response shows little <strong>com</strong>mand of basic elements of grammar or syntax, with numerous and frequent errors that makeideas in<strong>com</strong>prehensible.• The candidate demonstrates little knowledge of vocabulary beyond the most <strong>com</strong>mon words, and may misuse and mis<strong>for</strong>mnumerous words and expressions.• The candidate fails to maintain any flow of speech, with frequent hesitations and pauses that interrupt <strong>com</strong>prehensibility.• The speaker's pronunciation contains numerous errors and is at times unintelligible.The response is unrelated to the assigned topic, inaudible/in<strong>com</strong>prehensible, not spoken in the required language, not ofsufficient length to score, or merely a repetition of the assignment.There is no response to the assignment.65


Spanish (28) Practice TestFIRST SAMPLE WEAK RESPONSE FOR OPEN-RESPONSEITEM—WRITTEN EXPRESSIONQuerido señor,Primero, ¡Bienvenidos! Gracias para pedirnos preguntas sobre nuestra cultura. Tenemosmuchas museo muy cerca. Boston es 40 Kilometros y Hay mucho museos para visitar. Tambiénhay equipos de deportes, Los Red Sox, Celtics y Bruins son partidos se puede ver.Durante el verano, en julio, Tenemos fiestas y fuegos en el cielo. Es muy espetaculo. Estetiempo se marca nuestra independencia.Me gusta este tiempo por que Hace buen tiempo y los días están largo. El ultímo año fui aver los barcos muy viejos en Boston. Era un tiempo fenomenal.Un otro dato que te interesa es hay muchas universidades. voy a mostrar tú algunos, peromi colegio favorito es Harvard.Pues si tú tienes mas preguntas llamáme por favor.Sinceramente,[writer's name]ANALYSIS FOR FIRST WEAK RESPONSE TO OPEN-RESPONSEITEM—WRITTEN EXPRESSIONThis is an example of a weak response because it is characterized by the following:Purpose: The purpose is partially fulfilled because the examinee describes the culture of the Boston area in avery general and simplistic way. The ideas are too brief and disjointed. There is an attempt made to recount apersonal experience but it is too thin in description and weak in language <strong>for</strong> a <strong>com</strong>plete message. The letter lacksthe <strong>for</strong>mality of <strong>for</strong>m and language required by the situation and assignment.Coherence: The response exhibits limited organization by presenting distinct separate paragraphs. The readermust search <strong>for</strong> the connections between them. There is no plan stated or implied. The simplicity and vocabularyerrors interfere with the clarity of the ideas presented.Content: The ideas of the first paragraph lack any development and are in isolation from the rest of the response.The attempt to discuss summer activities is disjointed and is, in fact, an unsupported list (e.g., Independence Day,the weather, the tall ships). There is no connection or development in the paragraph about colleges and it is notrelated to the rest of the response.Grammar: The response demonstrates limited <strong>com</strong>mand. Ser and estar are misused: los días están largo,Boston es 40 Kilometros. Adjectives do not agree with nouns: muchas museo and mucho museas, los días estánlargo. There is a lack of subordination: es hay muchas universidades. The imperative is used incorrectly; it isfamiliar llamame. Direct object pronouns are not used and the present substitutes <strong>for</strong> the future: voy a mostrar tú.Vocabulary: The response lacks key words. Examples include: fuegos en el cielo, es muy espetaculo, pedirpreguntas, un otro dato, mi colegio favorito, tiempo <strong>for</strong> estación.Mechanics: The response has errors that partially interfere with <strong>com</strong>munication. Examples include: Kilometrosinstead of kilómetros, Tenemos instead of tenemos, espetaculo instead of espectáculo, por que instead of porque,voy a instead of Voy a, llamáme instead of llámame.66


Spanish (28) Practice TestSECOND SAMPLE WEAK RESPONSE FOR OPEN-RESPONSEITEM—WRITTEN EXPRESSIONHola y bienvenido a Nueva Hampshire.Va a encontrase en un sitio con mucho que hace. La clima en Nueva Hampshire es calienteen el verano y muy frio en el invierno. Este invierno recibimos muchos torrentes de nieve.Entonces, somos muy contentas hora que esta bueno tiempo afuera.Recreaciones aqui son muchos deportes dependiende en que tiempo es. En el inviernoesquiar es grande y en el verano nadar en el mar es grande.Puedes tambien dar paseos y correr por la calle. Aqui tenemos muchos animales y fincas.En las fincas hay: cerdos, pollos, caballos, ranas, y puerros.Cuando visitas nuestra escuela puedes encontra muchos libros en Español en nuestrabiblioteca. Nuestra escuela se da orgullo en tener muchos libros.Cuando vienes vas a conocer a los estudiantes y al principal de nuestra colegio. ¡Noprecupas! Ella es muy fascinada con concerte y a discusar la educacion. Nuestra escuela estaplaneando un celebracion para tu viaje acqui entonces no olvidas tu vestido guapo.¡Hasta pronto![writer's name]67


Spanish (28) Practice TestANALYSIS FOR SECOND WEAK RESPONSE TO OPEN-RESPONSEITEM—WRITTEN EXPRESSIONThis is an example of a weak response because it is characterized by the following:Purpose: The purpose of the assignment is only partially achieved in this response because the response lacks afull description of the culture of New Hampshire, relying on brief disjointed cultural highlights (e.g., weather,sports, rural and school life). There is no personal experience given. The response is too in<strong>for</strong>mal in tone andlanguage (e.g., hola, inconsistent tú <strong>for</strong>ms, hasta pronto). The response is not entirely appropriate <strong>for</strong> the requiredlevel of politeness.Coherence: The first two paragraphs attempt to <strong>com</strong>pare and contrast winter and summer activities. However,the reader is <strong>for</strong>ced to make that connection. The middle of the response attempts to <strong>com</strong>pare and contrast urbanand rural life, but the ideas are so confusing that it seems as if the farm animals are in the city streets. The lasttwo paragraphs attempt to describe the candidate's school and partially achieve that goal, although the ideas arenot connected to each other.Content: There is a weak attempt at development, but a lack of relevant supporting details. The overall effect isthat of an outline. The ideas present have little or no development (e.g., contrast of winter and summer, contrastof urban and rural life).Grammar: The response demonstrates limited <strong>com</strong>mand: use of infinitive (va a encontrase), esquiar instead ofel esquiar, mucho que hace, puedes encontra, use of per and estar (somas muy contentas); incorrect article-nounagreement (la clima, un celebracion). Only the present tense is used. All <strong>com</strong>mand <strong>for</strong>ms are incorrect (noprecupas, no olvidas) and weather expressions are also used incorrectly (esta bueno tiempo instead of hace buentiempo).Vocabulary: The response lacks key words. Examples include torrentes de nieve, hora instead of a hora, use ofgrande (nadar es grande), ranas y puerros, principal instead of director, discusar instead of discutir.Mechanics: The response has errors that partially interfere with <strong>com</strong>munication. Examples include frio insteadof frío, aqui instead of aquí, tambien instead of también, Español instead of español, educacion instead ofeducación, esta instead of está, and celebracion instead of celebración.68


Spanish (28) Practice TestFIRST SAMPLE STRONG RESPONSE FOR OPEN-RESPONSEITEM—WRITTEN EXPRESSIONEstimada colega,Esperamos con anticipación su llegada a <strong>Massachusetts</strong>. Hay mucho que hacer cerca deBoston; usted gozará de su visita y de conocer a mucha gente diversa y amistosa.Cambridge, una cuidad bastante pequeña pero internacional, ofrece una cultura diversa yuna historia larga e interesante. Se puede participar en una gran variedad de actividades cadadía.Las universidades de Harvard y M.I.T. <strong>for</strong>man el enfoque educativo que atraen a losmejores estudiantes por el mundo. Esta población estudiantil requiere que los restaurantesofrezcan un menú variado y diverso. Por eso, cada persona puede encontrar algo a su gusto y ala vez escuchar música típica de su país. Se refleja la diversidad de la gente también alcaminar por las calles. Siempre se ven a personas jóvenes e interesantes que dibujan escenasde la ciudad, que hacen mimo para divertir a los caminantes, y que tocan cualquier instrumentoque se puede llevar.Una vez mi novio catalán y yo buscábamos un queso especial para servir <strong>com</strong>o tapa con cava.Descubrimos la tienda Formaggio donde había un montón de quesos artesanales. Eldependiente sabio nos dejó probar unos cuantos hasta que decidimos <strong>com</strong>prar un Manchegosabroso de La Mancha.Para llegar a Cambridge, es fácil tomar el metro o el autobús, montar en bicicleta, ycaminar de varios sitios de Boston. Vivir cerca de una ciudad tan internacional es <strong>com</strong>o teneruna puerta al mundo.Atentamente,[writer's name]69


Spanish (28) Practice TestANALYSIS FOR FIRST STRONG RESPONSE TO OPEN-RESPONSEITEM—WRITTEN EXPRESSIONThis is an example of a strong response because it is characterized by the following:Purpose: This response thoroughly fulfills the purpose because the candidate gives an elaborated overview of thearea (Cambridge), describing several cultural activities, and provides a personal anecdote about a specificexperience. It is <strong>com</strong>pletely appropriate in tone, using consistent <strong>for</strong>mal language, and it is written in the <strong>for</strong>m ofa letter.Coherence: In a well-organized manner, the candidate presents a unified discussion of cultural diversity and theeveryday activities that reflect it. In the third paragraph, the examinee clearly describes the varied culturalofferings of the city (e.g., restaurants, food, music, sites). Ideas are linked by using transitions (esa, por eso,también).Content: The main idea, cultural diversity, as reflected in daily activity, is well supported, with rich, relevantsupporting detail. The college <strong>com</strong>munity's diversity is reflected not only in the cuisine, people, and music of thearea, but also in the variety of activities seen on the city's streets. The personal experience connects indirectlywith both the food and cultural diversity.Grammar: The response exhibits mastery of grammatical <strong>for</strong>ms and syntax. Adjective-noun phrases arecorrectly agreed and positioned: mucha gente diversa, personas jóvenes e interesantes. Subjects and verbs agree:Cambridge . . . ofrece; las universidades . . . <strong>for</strong>man. Use of the preterite-imperfect is correct: descubrimos,había, dejó, decidimos. Sentences are varied and subordination adds <strong>com</strong>plexity: Los universidades deHarvard . . . por el mundo. Direct objects are correctly placed: nos dejó. The subjunctive is used correctly:ofrezcan.Vocabulary: The response demonstrates a broad <strong>com</strong>mand and appropriate use of vocabulary. Examplesinclude gozar de, enfoque educativo, estudiantil, artesanales, hacen mimo, divertir, cualquier instrumento, unoscuantos. The adjective estudiantil, which follows esta población, is an appropriate descriptor <strong>for</strong> this level. Alower-level response may have included de estudiantes.Mechanics: The response exhibits mastery of spelling and diacritical marks, whether they are used aspronunciation aids (anticipación) or tense indicators (gozará).70


Spanish (28) Practice TestSECOND SAMPLE STRONG RESPONSE FOR OPEN-RESPONSEITEM—WRITTEN EXPRESSIONDistinguido profesor Suárez,La presente es para in<strong>for</strong>marle sobre el área donde vivo para que usted conozca <strong>com</strong>o esmi <strong>com</strong>unidad. La cultura de Springfield es bien variada porque aquí viven personas de variaspartes del mundo incluyendo hispanoparlantes, chinas, italianas, y rusas.Aunque existen días de fiesta típicos de varias nacionalidades, hay un gran número deactividades populares cuyo origen se basa en la tradición americana.En invierno, hace mucho frío y la gente prefiere quedarse en casa mirando la televisión oleyendo la novela más popular del día. Durante el mes de diciembre, muchas familias dan unpaseo en auto por el parque “Forest” para ver la famosa festival de luces.La primavera nos trae unos días más templados y <strong>com</strong>o las mariposas del museo del mismonombre, la gente sale a explorar su alrededor. Los aficionados de deportes visitan laexhibición de baloncesto donde observan los recuerdos y los trofeos de los más famosos. Elmuseo del autor celebre Doctor Seuss atrae a los niños, a sus padres y a sus maestros. Lasfamilias también visitan el museo de Eric Carle para crear obras artísticas infantiles.En verano, la rutina diaria se hace más saludable <strong>com</strong>o todos visitan los muchos parques de<strong>com</strong>unidad. Uno se puede caminar con el perro, hacer jogging, volar una <strong>com</strong>eta, o participaren un sinnúmero de deportes.La gran migración estudiantil a las varias universidades cercanas, junto con latrans<strong>for</strong>mación del color de las hojas de árboles, caracterizan el otoño. Los habitantesrecuerdan sus raíces campestres cuando asisten a la "Grande E" para ver y <strong>com</strong>prar lo queofrecen los artesanos y los agricultores.Recuerdo con gran afición las visitas familiares a la granja local para escoger una calabazaenorme. Al llegar a casa mi papá siempre me ayudaba a diseñar una cara espantosa en la pielde la fruta.Ojalá que usted encuentre útil la in<strong>for</strong>mación de esta carta. En espera de sus noticias desu llegada, quedo muy atentamente.[writer's name]71


Spanish (28) Practice TestANALYSIS FOR SECOND STRONG RESPONSE TO OPEN-RESPONSEITEM—WRITTEN EXPRESSIONThis is an example of a strong response because it is characterized by the following:Purpose: The candidate provides a thorough description of the area, using the seasons of the year, relatedweather expressions, and activities as a base to provide cultural in<strong>for</strong>mation. A brief but relevant personalexperience is included. The response's <strong>for</strong>m is <strong>com</strong>pletely appropriate and includes a proper salutation andclosing while maintaining correct and <strong>for</strong>mal language.Coherence: The writing plan of the response is clearly expressed in the second paragraph: popular traditionalAmerican activities as a reflection of culture. The plan is thoroughly achieved in the next four paragraphs, usingthe seasons as a unifying canvas.Content: By enumerating the seasonal activities with rich detail and variety, the writer extensively develops themain idea. Examples include the light festival in Forest Park; the several museums of the area, each of a singularconcept; and the typical summer and autumnal activities.Grammar: The response exhibits mastery of grammatical <strong>for</strong>ms and syntax. Adjectives and nouns showappropriate agreement and positioning: días de fiesta típicos, la gran migración estudiantil, la rutina diaria.Subjects and verbs agree: la gente prefiere, aquí viven personas. Complex structures include the use of thesubjunctive after para que (conozca) and ojalá que (encuentre) and the use of preterite-imperfect: mi papásiempre me ayudaba. Sentences are varied and subordinated.Vocabulary: The response demonstrates a broad <strong>com</strong>mand and appropriate use. Examples includehispanoparlantes, templados, obras artísticas infantiles, un sinnúmero, caracterizan, raíces campestres, granafición, and granja local.Mechanics: Spelling, punctuation, and diacritical marks are consistently accurate, contributing clarity to theresponse.72


Spanish (28) Practice TestDESCRIPTION OF A STRONG RESPONSE FOR OPEN-RESPONSEITEM—ORAL EXPRESSIONA strong response to this assignment would address the topic of hobbies or pastimes fully and offer advice to afriend within the provided context in coherent, appropriate Spanish. Ideas presented should be bolstered byelaboration (specifying details, extended exploration of topics/assertions, relevant reasoning) that deepens themand fleshes out the response. The sequencing of ideas should be logical and transitions between ideas smooth.The oral language should be fluid and clear, with accurate pronunciation and a consistent flow of speech thatdisplays facility with Spanish. Pauses in the response should be appropriate to the delivery of the content and notthe result of uncertainty about the language itself or the inability to pronounce words or construct sentences.Ideas should be expressed with phrases and grammatical constructions that are true to the Spanish language andnot imported from another language. A variety of relevant vocabulary words, expressions, verb tenses andmoods, and syntactical constructions should be employed in order to render the response clear and precise. Theresponse should be free from grammar and usage errors.73


Spanish (28) Practice TestPRACTICE TEST SCORE CALCULATIONThe practice test score calculation is provided so that you may better gauge your per<strong>for</strong>mance and degree ofreadiness to take an <strong>MTEL</strong> test at an operational administration. Although the results of this practice test may beused as one indicator of potential strengths and weaknesses in your knowledge of the content on the official test, itis not possible to predict precisely how you might score on an official <strong>MTEL</strong> test.The Sample Responses and Analyses <strong>for</strong> the open-response items may help you determine whether yourresponses are more similar to the strong or weak samples. The Scoring Rubric can also assist in estimating ascore <strong>for</strong> your open responses. You may also wish to ask a mentor or teacher to help evaluate your responses tothe open-response questions prior to calculating your total estimated score.How to Calculate Your Practice Test ScoreReview the directions in the sample below and then use the blank practice test score calculation worksheet on thefollowing page to calculate your estimated score.Multiple-Choice SectionSAMPLEEnter the total number of multiple-choice questions you answered correctly: 74Use Table 1 below to convert that number to the score and write your score in Box A: A: 170Open-Response SectionEnter the number of points (1 to 4) <strong>for</strong> your first open-response question: 3Enter the number of points (1 to 4) <strong>for</strong> your second open-response question: 3======Add those two numbers (Number of open-response question points): 6Use Table 2 below to convert that number to the score and write your score in Box B: B: 72Total Practice Test Score (Estimated <strong>MTEL</strong> Score)Add the numbers in Boxes A and B <strong>for</strong> an estimate of your <strong>MTEL</strong> score: A + B = 24274


Spanish (28) Practice TestPractice Test Score Calculation Worksheet: SpanishTable 1: Table 2:Number ofMultiple-ChoiceQuestions CorrectEstimated<strong>MTEL</strong>ScoreNumber ofMultiple-ChoiceQuestions CorrectEstimated<strong>MTEL</strong>ScoreNumber ofOpen-ResponseQuestion PointsEstimated<strong>MTEL</strong>Score0 to 25 86 61 to 65 150 2 3626 to 30 94 66 to 70 158 3 4531 to 35 102 71 to 75 170 4 5436 to 40 110 76 to 80 174 5 6341 to 45 118 81 to 85 183 6 7246 to 50 126 86 to 90 191 7 8151 to 55 134 91 to 95 199 8 9056 to 60 142 96 to 100 207Print the <strong>for</strong>m below to calculate your estimated practice test score.Multiple-Choice SectionEnter the total number of multiple-choice questions you answered correctly:Use Table 1 above to convert that number to the score and write your score in Box A: A:Open-Response SectionEnter the number of points (1 to 4) <strong>for</strong> your first open-response question:Enter the number of points (1 to 4) <strong>for</strong> your second open-response question:Add those two numbers (Number of open-response question points):======Use Table 2 above to convert that number to the score and write your score in Box B: B:Total Practice Test Score (Estimated <strong>MTEL</strong> Score)Add the numbers in Boxes A and B <strong>for</strong> an estimate of your <strong>MTEL</strong> score: A + B =75

Hooray! Your file is uploaded and ready to be published.

Saved successfully!

Ooh no, something went wrong!